Sunteți pe pagina 1din 71

GATE 2015 Solved Paper

Gratitude Aptitude Test in Engineering


Set − 1
Number of Questions: 65 Total Marks: 100.0

Wrong answer for MCQ will result in negative marks, (−1/3) for 1 mark Questions and (−2/3) for 2 marks Questions.

General Aptitude
Number of Questions: 10 Selection Marks: 15.0
Q.1 to Q.5 carry 1 mark each and Q.6 to Q.10 carry 2 If two floors in a certain building are 9 feet apart, how many
marks each. steps are there in a set of stairs that extends from the first
Question Number: 1 Question Type: MCQ floor to the second floor of the building?
Didn’t you buy _________ when you went shopping? Statements:
3
(A) any paper (B) much paper (I) Each step is foot high.
(C) no paper (D) a few paper 4
(II) Each step is 1 foot wide.
Solution:  The use of ‘any paper’ is correct. The use of (A) Statement I alone is sufficient but statement II
‘a few’ would have been correct had it been followed by alone is not sufficient.
‘papers’ and ‘not paper’. The rest of the choices render the (B) Statement II alone is sufficient, but statement I
sentence incorrect. alone is not sufficient.
Hence, the correct option is (A). (C) Both statements together are sufficient, but neither
statement alone is sufficient.
Question Number: 2 Question Type: MCQ
(D) Statements I and II together are not satisfied.
Which of the following options is the closest in meaning to
Solution:  The distance between the 2 floors of the building
the sentence below?
is 9 feet. And we are asked to find the number steps from
She enjoyed herself immensely at the party.
first floor to second floor.
(A) She had a terrible time at the party.
From I:
(B) She had a horrible time at the party.
If each step is 3/4th foot .
(C) She had a terrific time at the party
Let there be a total of n steps from first floor to record floor.
(D) She had a terrifying time at the party.
3
Solution:  The question statement means that the girl had ×n=9
4
a very good time at the party. Choice (C) brings this out.
4×9
‘Terrific’ means extremely good and is used in a positive n= = 12
connotation. The other choices have negative contexts. 3
So, I alone gives this answer
Hence, the correct option is (C). II speaks about the width of each step. From width we can-
Question Number: 3 Question Type: MCQ not get the number of steps between 1st and 2nd floors.
Which one of the following combinations is incorrect? Hence, the correct option is (A).
(A) Acquiescence − Submission
(B) Wheedle − Roundabout Question Number: 5 Question Type: MCQ
(C) Flippancy − Lightness Given Set A = {2, 3, 4, 5} and Set B = {11, 12, 13, 14,
(D) Profligate − Extravagant 15}, two numbers are randomly selected, one from each set.
Solution:  ‘Acquiescence’ means ‘submission’. ‘Wheedle’ What is the probability that the sum of the two numbers
is to persuade someone to do something for you. ‘Round equals 16?
about’ means indirect. ‘Flippancy’ means a lack of serious- (A) 0.20 (B) 0.25
ness in grave matters. ‘Lightness’ means the same. ‘Profli- (C) 0.30 (D) 0.33
gate’ means ‘extravagant’. Thus, choice (B) is the incorrect
Solution:  Let the numbers randomly selected from set A
option.
and set B be a and b respectively.
Hence, the correct option is (B). The number of (a, b) that can be formed taking a from A and
Question Number: 4 Question Type: MCQ b from B is 4 × 5 = 20. And the (a, b) for which a + b = 16
Based on the given statements, select the most appropriate are (2, 14), (3, 13), (4, 12), (5, 11).
option to solve the given question. Number of favourable selections = 4
2 | GATE 2015 Solved Paper Set − 1

4
Required probability = = 0.2
20
Hence, the correct option is (A). Electrical
20%
Question Number: 6 Question Type: MCQ Computer
Select the alternative meaning of the underlined part of the Science
sentence. Mechanical 40%
The chain snatchers took to their heels when the police 10%
party arrived.
(A) took shelter in a thick jungle
Civil
(B) open indiscriminate fire 30%
(C) took to flight
(D) unconditionally surrendered

Solution:  Took to their heels’ means to run away. This sup- Solution:  Number of students in the Electrical Engineer-
ports choice (C). The other answer choices are incorrect. 9
Hence, the correct option is (C). ing department = 40   = 72
5
Question Number: 7 Question Type: MCQ Number of students in the Civil department
The given statement is followed by some courses of action. 30
= (72) = 108
Assuming the statement to be true, decide the correct option. 20
Statement: Number of students in the Mechanical department
There has been significant drop in the water level in the 10
= (72) = 36
lakes supplying water to the city. 20
Course of action: Number of female students in the Civil and the Mechanical
(I) The water supply authority should impose a partial cut 4 4
departments are 108   and 36   respectively
in supply to tackle the situation. 9 9
(II) The government should appeal to all the residents
i.e. 48 and 16 respectively.
through mass media for minimal use of water.
Difference is 48 − 16 i.e., 32.
(III) The government should ban the water supply in lower
areas. Hence, the correct answer is 32.
(A) Statements I and II follow. Question Number: 9 Question Type: MCQ
(B) Statements I and III follow. The probabilities that a student passes in Mathematics,
(C) Statements II and III follow. Physics and Chemistry are m, p and c respectively. Of
(D) All statements follow. these subjects, the student has 75% chance of passing in
Solution:  When there is a significant drop in the water at least one, a 50% chance of passing in at least two and a
level in the lakes supplying water in the city. The plausible 40% chance of passing in exactly two. Following relations
course of action has to be the ones which are practically are drawn in m, p, c:
possible. (I)  p + m + c = 27/20
Among the three given courses of action, only I and II are (II)  p + m + c = 13/20
practically possible. III speaks about banning the water (III) (p) × (m) × (c) = 1/10
supply in lower areas. (A) Only relation I is true.
This is not an appropriate solution to the existing problem. (B) Only relation II is true.
And stopping or banning water in the lower areas for proper (C) Relations II and III are true.
supply in the city is unethical as well. (D) Relations I and III are true.
Hence, the correct option is (A). Solution:  VD for probabilities
Total = 1
Question Number: 8 Question Type: MCQ p m
The pie chart below has the breakup of the number of stu-
dents from different departments in an engineering college d b
for the year 2012. The proportion of male to female students g
f
in each department is 5:4. There are 40 males in Electrical
Engineering. What is the difference between the numbers c
of female students in the Civil department and the female
c
students in the Mechanical department?
GATE 2015 Solved Paper Set − 1  |  3

p + m + c = a + b + c + 2 (d + e + f ) +3g = (a + b + c + d + Answered Answered Not


e + f + g) + (d + e + f + 2g) Q.No. Marks Correctly Wrongly Attempted
75 40 20 27 1 2 21 17 6
= + + =
100 100 100 20 2 3 15 27 2

⇒ I is true and II is not true. 3 1 11 29 4

(p) (m) (c) = probability (The student passing in all the three 4 2 23 18 3

50 40 10 1 5 5 31 12 1
subjects) = − = =
100 100 100 10 What is the average of the marks obtained by the class in
Hence, I and III are true. the examination?
Hence, the correct option is (D). (A) 2.290 (B) 2.970
(C) 6.795 (D) 8.795
Question Number: 10 Question Type: MCQ Solution:  Average of the marks obtained by the class
The number of students in a class who have answered cor- 2( 21) + 3(15) + 1(11) + 2( 23) + 5(31) 299
rectly, wrongly, or not attempted each question in an exam, = = = 6.795
Total number of students 44
are listed in the table below. The marks for each question are
also listed. There is no negative or partial marking. Hence, the correct option is (C).

Electrical Engineering
Number of Questions: 55 Section Marks: 85.0
Q.11 to Q.35 carry 1 mark each and Q.36 to Q.65 carry 2 Marks each.
Question Number: 11 Question Type: NAT ∞
2
A random variable X has probability density function f(x) i.e., ∫−∞ xf ( x)dx = 3
as given below:
0 1 ∞
a + bx; for 0 < x < 1 2
f ( x) =  ⇒  ∫ x × 0 dx + ∫0 x × (a + bx)dx + ∫ x × 0dx =
3
 0; otherwise −∞ 1

If the expected value E[X] = 2/3, then Pr[X < 0.5] is 1


2
⇒  ∫ (ax + bx )dx =
2
_______.
0
3
Solution:  Given the probability density function of a ran-
dom variable X is 1
ax 2 bx 3  2
a + bx for 0 < x < 1 ⇒  +  = 3
f ( x) =  2 3 0
0 otherwise
We know that for any probability density function f(x) of a a b 2
⇒  + =
random variable X, 2 3 3

⇒ 3a + 2b = 4  (2)

−∞
f ( x )dx = 1
Solving (1) and (2), we get a = 0 and b = 2
0 1 ∞ \  f(x) becomes
⇒ ∫ 0dx + ∫ (a + bx)dx + ∫ 0dx = 1 2 x for 0 < x < 1
−∞ 0 1 f(x) = 
1 0 otherwise
⇒ ∫ (a + bx)dx = 1
0
0.5

1
Now P(x < 0.5) = ∫ f ( x )dx
bx  2 −∞
⇒ ax +  =1
2 0 0 0.5

b
= ∫ 0dx + ∫ 2 xdx
−∞ 0
⇒  a + =1
2 0.5
= x 2 
⇒ 2a + b = 2 (1) 0

2 =(0.5) 2
Given the expected value of X = E (X) = =0.25.
3
Hence, the correct answer is (0.25).
4 | GATE 2015 Solved Paper Set − 1

Question Number: 12 Question Type: NAT Solution: 


If a continuous function f(x) does not have a root in the ∂ 2 1 ∂ 1 ∂
interval [a, b], then which one of the following statements ∇f = (r fr ) + ( fθ sin θ ) + ( fθ sin θ )
∂r r sin θ ∂θ r sin θ ∂θ
is TRUE? 1 ∂
(A) f(a) ⋅ f(b) = 0 + ( f◊ )
r sin θ ∂ ◊
(B) f(a) ⋅ f(b) < 0
1 ∂ 2 1
(C) f(a) ⋅ f(b) > 0 ∇f = (r × 2 ) = 0
r 2 ∂r r
(D) f(a)/f(b) ≤ 0
Hence, the correct option is (A).
Solution:  As f(x) is continuous in [a, b] and f(x) has no root
in [a, b], f(x) does not cut x-axis for all x in [a, b] Question Number: 15 Question Type: NAT
⇒  f(x) is either above x-axis or below x-axis for both When the Wheatstone bridge shown in the figure is used to
x = a and x = b find the value of resistor Rx, the galvanometer G indicates
⇒  f(a) and f(b) are both positive or both negative zero current when R1 = 50 Ω, R2 = 65 Ω and R3 = 100 Ω. If
⇒  f(a) ⋅ f(b) > 0. R3 is known with ±5% tolerance on its nominal value of 100
Hence, the correct option is (C). Ω, what is the range of Rx in Ohms?
Question Number: 13 Question Type: NAT
R1 R2
If the sum of the diagonal elements of a 2 × 2 matrix is
-6, then the maximum possible value of determinant of the
matrix is _______. G
Solution:
Let A be a 2 × 2 matrix with the sum of the diagonal R3 Rx
elements as − 6
Let l1 and l2 be the eigen values of A
\  The sum of the diagonal elements of A = −6 V
+ −
⇒  l1 + l2 = − 6 (1)
Det of A = |A| = l1l2
Now we have to find the maximum value of l1l2 (A) [123.50, 136.50] (B) [125.89, 134.12]
Let f = l1l2 (C) [117.00, 143.00] (D) [120.25, 139.75]
= l1 (−6 − l1) (from (1)) Solution:  Given
\  f = −6l1 − λ12
R1 R2
⇒  f ′ = −6 −2l1
For f to have maximum, f ′ = 0
⇒  − 6 − 2l1 = 0
⇒  l1 = − 3
Now f ″ = − 2 < 0 R3 Rx
\  f has a maximum at l1 = − 3
From (1), l1 + l2 = − 6
⇒  −3 + l2 = − 6
⇒  l2 = − 3
The maximum value of the determinant of A = l1l2 + −
V
= (− 3) × (− 3) = 9.
Hence, the correct answer is 9. R1 = 50 Ω
Question Number: 14 Question Type: NAT R2 = 65 Ω
→ 1 R3 = 100 Ω
Consider a function f = rˆ , where r is the distance from R3 tolerance ± 5%
r2
Then R3 = 95, ⇒105
the origin and r̂ is the unit vector in the radial direction. The
When bridge is balanced R1 Rx = R2 R3
divergence of this function over a sphere of radius R, which
When R3 = 95 ⇒ 50 Rx = 65 × 95
includes the origin, is
(A) 0 (B) 2π 65 × 95
Rx = = 123.5 Ω
(C) 4π (D) Rπ 50
GATE 2015 Solved Paper Set − 1  |  5

When R3 = 105 ⇒ 50 Rx = 65 × 105 S L

65 ×105
Rx = = 136.5 Ω
50 + 3Ω
20 V − C
Hence, the correct option is (A). +
5V
Question Number: 16 Question Type: NAT
A(0 − 50 A) moving coil ammeter has a voltage drop of 0.1
Solution:  given data duty ratio δ = 0.4
V across its terminals at full scale deflection. The external
input voltage = Vi = 20 V
shunt resistance (in milliohms) needed to extend its range to
battery voltage E = 5 V
(0 − 500 A) is ________.
Resistance R = 3 Ω
Solution:  Voltage drop of (0 − 50 A) ammeter is = 0.1 V Output voltage Vo = δVi
External shunt resistance Rsh = ? Vo = 0.4 × 20 = 8 V
V −E 3
Extend range − (0 − 500 A) Charging current I = o = = 1A
I = 50 A R 3
Im = 500 A Hence, the correct answer is 1.
Voltage drop I Rm = 0.1 Question Number: 19 Question Type: NAT
t
0.1 1
T t −∫T
Rm = = 2 mΩ A moving average function is given by y(t) = u(τ ) dτ.
50
Rm 1
Shunt resistance Rsh = If the input u is a sinusoidal signal of frequency Hz,
 I m   2T
 I  −1 then in steady state, the output y will lag u (in degree) by
  
______.
2 ×10 −3
= Solution:  u(τ) = sin ωτ
 500  
 50  −1 1
t t −τ
  
sin(ωτ ) dτ = cos(ωτ )
y(t) = ∫
T t −T ωτ t
=2.22 × 10-4
=0.22 mΩ 1
= [cos ω(t − τ) − cos ωt]
Hence, the correct answer is 0.22 to 0.23. π
Question Number: 17 Question Type: NAT 2 2
y(t) = - cos ωt = sin(90 + ωt)
Of the four characteristics given below, which are the major p p
requirements for an instrumentation amplifier? y(t) will lag u by 90°
P. High common mode rejection ratio
Hence, the correct answer is 90°.
Q. High input impedance
R. High linearity Question Number: 20 Question Type: NAT
S. High output impedance The impulse response g(t) of a system G, is as shown in
(A) P, Q and R only Figure (a). What is the maximum value attained by the
(B) P and R only impulse response of two cascaded blocks of G as shown in
(C) P, Q and S only Figure (b)?
(D) Q, R and S only
Solution:  Instrumentation amplifier has to amplify small
changes in the bridge circuit. So, for ideal instrumentation g(t )
amplifier high CMMR, high, input impedance, high, linear- 1
ity and low output impedance required.
Hence, the correct option is (A).
Question Number: 18 Question Type: NAT t
0 1
In the following chopper, the duty ratio of switch is 0.4. If   (a)
the inductor and capacitor are sufficiently large to ensure
continuous inductor current and ripple free capacitor volt- G G
age, the charging current (in Ampere) of the 5 V battery,
  (b)
under steady-state is ______.
6 | GATE 2015 Solved Paper Set − 1

2 3 Question Number: 22 Question Type: NAT


(A) (B)
3 4 A steady current I is flowing in the −x direction through
L
4 each of two infinitely long wires at y = ± as shown in the
(C) (D) 1 2
5 →
figure. The permeability of the medium is µ0. The B -field
Solution:  Given h(t) = g(t) × g(t) at (0, L, 0) is
The h(t) wave form
Z

0 1 2 Y = −L /2 Y = L /2
Y
So maximum value of cascaded response is 1. 0
Current = l Current = l
Hence, the correct option is (D).
Question Number: 21 Question Type: NAT X
Consider a one-turn rectangular loop of wire placed in a
uniform magnetic field as shown in the figure. The plane of 4µ t 4µ I
+ 0 Ẑ
− 0 Ẑ (B)
(A)
the loop is perpendicular to the field lines. The resistance 3π L 3π L
of the loop is 0.4 Ω, and its inductance is negligible. The 3µ 0 I
magnetic flux density (in Tesla) is a function of time, and (C) 0 (D) − Ẑ
4π L
is given by B(t) = 0.25 sin ωt, where ω = 2π × 50 radian/
second. The power absorbed (in Watt) by the loop from the Solution:  H = H1 + H2
magnetic field is _____. I I
= ( − az ) + ( −aZ )
2π ( L 2)  3L 
10 cm 2π  
 2 

4I
= (− az); B = µoH
3π L
5 cm Hence, the correct option is (A).
Question Number: 23 Question Type: NAT
Consider the circuit shown in the figure. In this circuit R =
1 k Ω, and C = 1 µF. The input voltage is sinusoidal with a
frequency of 50 Hz, represented as a phasor with magni-
Solution:  Resistance of Loop R = 0.4 Ω tude Vi and phase angle 0 radian as shown in the figure. The
B(t) = 0.25 sin ωt output voltage is represented as a phasor with magnitude Vo
ω = 2π × 50 radian/second and phase angle δ radian. What is the value of the output
phase angle δ(in radian) relative to the phase angle of the
1
flux f = ∫ B.ds = sin ωt input voltage?
800
R
dϕ −1
V=− = π cos ωt C
dt 8 −
V π
2 2 Vi = Vi ∠0 Vo = Vo ∠δ
P= = cos 2 ωt +
R 64 R C
R
π 2  1 + cos 2ωt 
P=
0.4 × 64  2 

π2 (A) 0 (B) π
Pavg = = 0.193
2 × 0.4 × 64 π (D) π
(C) −
Hence, the correct answer is 0.193. 2 2
GATE 2015 Solved Paper Set − 1  |  7

Solution:
I0
R
I1 4×1
C MUX F
Vi = Vi ∠0 I2

∼ 1
C I3
+ V o = V o ∠δ S1 S0

R
A B

(A) 1010 (B) 0110


(C) 1000 (D) 1110
By considering virtual Grounding concept the nodal
Solution:  F = A + B = AB + AB
0 −Vi ∠0 o 0 −Vo ∠δ
equation at node (1) is + =0 S1 S0
XC R
Vo ∠ δ = -j ωRC Vi ∠ 0° A B - 0 - Io
δ = -90° A B - 1 − I1
Hence, the correct option is (D). A B - 1 − I2
Question Number: 24 Question Type: NAT A B - 0 − I3
In the given circuit, the silicon transistor has β = 75 and a Hence, the correct option is (B).
collector voltage VC = 9 V. Then the ratio of RB and RC is
Question Number: 26 Question Type: NAT
______.
Consider a HVDC link which uses thyristor based line-
15 V commutated converters as shown in the figure. For a power
flow of 750 MW from system 1 to System 2, the voltages at
the two ends, and the current, are given by: V1 = 500 kV, V2
= 485 kV and I = 1.5 kA. If the direction of power flow is
to be reversed (that is, from System 2 to System 1) without
RC changing the electrical connections, then which one of the
RB following combinations is feasible?
VC System 1 I System 2
+ +
V1 V2
− −

V1 = -500 kV, V2 = -485 kV and I = 1.5 kA


(A)
V1 = -485 kV, V2 = -500 kV and I = 1.5 kA
(B)
V1 = 500 kV, V2 = 485 kV and I = -1.5 kA
(C)
Solution:  Given b = 75; VC = 9 V V1 = -500 kV, V2 = -485 kV and I = -1.5 kA
(D)
Collector equation ⇒ 15 − 9 − (IC + IB)RC = 0 Solution:  For reversing power flow in HVDC system the
76 IB RC = 6 (1) direction of current same.
Base ⇒ 15 − 76 IB RC − IB RB − 0.7 = 0 V −V
I= 1 2
⇒ 15 − 6.7 = IB RB R
Option (A): V1 = −500 kV, V2 = -485 kv
⇒  IB RB = 8.3 (2)
I = -1.5 kA
( 2) RB 8.3 × 76 Option (B): V1 = − 485 kV, V2 = − 500 kV
= = 105.133
(1) RC 6 I = 1.5 kA
Hence, the correct option is (B).
Hence, the correct answer is 105.133.
Question Number: 27 Question Type: NAT
Question Number: 25 Question Type: NAT Base load power plants are
In the 4 × 1 multiplexer, the output F is given by F = A + B. P: wind farms.
Find the required input ‘I3 I2 I1 I0’. Q: run-of-river plants.
8 | GATE 2015 Solved Paper Set − 1

R: nuclear power plants. Solution:  Given


S: diesel power plants. 20 i
(A) P, Q and S only 1Ω
− + A
(B) P, R and S only i1 +
(C) P, Q and R only i
+
(D) Q and R only 2V 1Ω 2 Ω V th

Solution:  Diesel power plants are peak load plants and −
wind plants are base load plants because once wind plants B
comes into operation it used for all times because no fuel
cost. Vth = 2i1
Hence, the correct option is (C). 2 = 1(i + i1) + i
Question Number: 28 Question Type: NAT 2 = 2i + i1  (1)
The voltages developed across the 3 Ω and 2 Ω resistors 21
21i = 2i1  ⇒  i1 = i (2)
shown in the figure are 6 V and 2 V respectively, with the 2
polarity as marked. What is the power (in Watt) delivered by
Solving (1) and (2)  i1 = 1.68 A
the 5 V voltage source?
Vth = 2i1 = 2 × 1.68 = 3.36 V
6V
− +
Hence, the correct option is 3.36 V.

2Ω Question Number: 30 Question Type: NAT
Network N1 Network N2 An inductor is connected in parallel with a capacitor as
+ 2V −
shown in the figure.
− +
5V i L
(A) 5 (B) 7 C
Z
(C) 10 (D) 14
6
Solution:  Given I3 = =2 As the frequency of current i is increased, the impedance (Z)
3
of the network varies as
2
I2 = =1 (A)
2
Inductive
− 6V + I3
Z
f

N1 N2

I2 + 2 V capacitive

−+
5V I Inductive

I + I2 = I3
I + 1 = 2  ⇒  1 A Z
f
Power of voltage source P = VI = 5 W
Hence, the correct option is (A). Capacitive
Question Number: 29 Question Type: NAT (B)
For the given circuit, the Thevenin equivalent is to be (C)
Inductive Capacitive
determined. The Thevenin voltage, VTh (in Volt), seen from
terminal AB is ______. Z
20 i f

− + A
(D)
+ i Z Inductive
2V 1Ω 2Ω
f
Capacitive
B
GATE 2015 Solved Paper Set − 1  |  9

1 Solution:  L1 = 800 mH
jω L ×
Z L ZC jω c  L2 = 600 mH
Solution  Z = =  M = 480 mH
Z L + ZC 1
jω L +
jω c  ω2 M 2 
jXL =  jX 1 + 
 j X 22 
 ωL 
Z= j   
Leff = 314 × 0.8 − 314 × 0.48  314
2
1 − ω LC 
2
  
 0.6 × 314 
For different frequencies the curves similar to choice (B).
Question Number: 31 Question Type: NAT 130.744
=
A separately excited DC generator has an armature 314
resistance of 0.1 Ω and negligible armature inductance. At Leff = 416 mH
rated field current and rated rotor speed, its open-circuited
voltage is 200 V. When this generator is operated at half the Hence, the correct option is (A).
rated speed, with half the rated field current, an uncharged Question Number: 33 Question Type: NAT
1000 µF capacitor is suddenly connected across the armature The primary mmf is least affected by the secondary terminal
terminals. Assume that the speed remains unchanged conditions in a
during the transient. At the time (in microsecond) after the (A) power transformer
capacitor is connected will the voltage across it reach 25 V? (B) potential transformer
(A) 62.25 (B) 69.3 (C) current transformer
(C) 73.25 (D) 77.3 (D) distribution transformer
Solution:  Armature resistance Ra = 0.1 Ω Solution:  In current transformer least number of second-
rated field current and rated rotor speed open circuit voltage ary turns, therefore, the effect on primary mmf is least.
E1 = 200 V Hence, the correct option is (C).
With half-rated field current and half-rated speed
Question Number: 34 Question Type: NAT
φ N
i.e., 1 & 1 A Bode magnitude plot for the transfer function G(s) of a
2 2 plant is shown in the figure. Which one of the following
Capacitance C = 1000 µF transfer functions best describes the plant?
E  N
20 log|G(j2pf)|
E1 φ1 N1
⇒ =
E 2 φ2 N 2 20
0
200 φ N
⇒ = 1 1 ⇒ E2 = 50 V −20
E2 φ1 N1
×
2 2 0.1 1 10 100 1 k 10 k 100 k f (Hz)
τ = RC = 0.1 × 1000 × 10-6
1000( s + 10)
E = 2000 e-t/RC (A)
−6 s + 1000
50 = 2000 e − t /( 0.1×1000 ×10 )
t = 69.3 µs 10( s + 10)
(B)
Hence, the correct option is (B). s( s + 1000)
Question Number: 32 Question Type: NAT s + 1000
The self inductance of the primary winding of a single phase, (C)
10 s( s + 10)
50 Hz, transformer is 800 mH, and that of the secondary
winding is 600 mH. The mutual inductance between these s + 1000
two windings is 480 mH. The secondary winding of this (D)
10( s + 10)
transformer is short circuited and the primary winding
is connected to a 50 Hz, single phase, sinusoidal voltage Solution:  From the given bode plot one pole at -10 and
source. The current flowing in both the windings is less one zero at 1000. The general transfer function is
than their respective rated currents. The resistance of both  S 
k 1 +
1000 
windings can be neglected. In this condition, what is the
effective inductance (in mH) seen by the source? G ( s) = 
(A) 416 (B) 440  S 
1 + 10 
(C) 200 (D) 920  
10 | GATE 2015 Solved Paper Set − 1

Finding k ÷ The initial slope 20 log k = 20 The characteristic equation of A is |A − lI| = 0


⇒  K = 10 −3 − λ 0 −2
 S  1 −1 − λ 0 =0
10 1 +
1000  S + 1000
G ( s) =  = 0 a −2 − λ
S 10( S + 10)
1+ ⇒ (− 3 − l) (− 1 − l) (− 2 − l) − 2a = 0
10
⇒ (l + 1) (l + 2) (l + 3) + 2a = 0 (1)
Hence, the correct option is (D). We know that if A has three district Eigen values then A has
Question Number: 35 Question Type: NAT three linearly independent eigen vectors
For the signal-flow graph shown in the figure, which one of Let f(l) = (l + 1) (l + 2) (l + 3)
the following expressions is equal to the transfer function \  (1) becomes,
f(l) + 2a = 0
Y ( s) ⇒  f(l) = −2a(2)
?
X 2 ( s) X1 ( s ) = 0
Consider f(x) = (x + 1) (x + 2) (x + 3)
The graph of f(x) is as shown in the figure below
X 1(s) X 2(s) Y

Q (x2,y2)

1 G1 G2
Y (s)

−3 −2 −1 0 X
−1 −1

G1 G y = f (x) P(x1,y1)
(A) (B) 2
1 + G2 (1 + G1 ) 1 + G1 (1 + G2 )
G G2 We know that the number of distinct real roots of an equation
(C) 1 (D) F(x) = k, (k is real) is same as that of the number of points of
1+ G1G2 1+ G1G2
intersection of the curve y = F(x) and the line y = k
Y ( s) The curve y = f(x) intersects at three points with a line y = y0
Solution:  The forward path given for is G2 only when y1 ≤ y0 ≤ y2
X 2 (S ) X1 ( S ) = 0
i.e., for f(x) + 2a = 0 (OR) f(x) = − 2a, three distinct real
By using masons gain formula roots exist for
G2 G2 y1 ≤ −2a ≤ y2(3)
The transfer function = =
1+ G1 G2 + G2 1 + G2 (1 + G1 ) i.e., y1 ≤ f(x) ≤ y2 (4) (from (2))
Now we will find y1 and y2 (i.e., the minimum and maximum
Hence, the correct option is (B). values of f(x))
Question Number: 36 Question Type: NAT f(x) = (x + 1) (x + 2) (x + 3) = x3 + 6x2 + 11x + 6
The maximum value of “a” such that the matrix ⇒  f ′(x) = 3x2 + 12x + 11
f ′(x) = 0  ⇒ 3x2 + 12x + 11 = 0
 −3 0 −2 
  −6 ± 3
 1 −1 0  has three linearly independent real ⇒  x =
3
 0 a −2 
  And f ″(x) = 6x + 12
eigenvectors is
−6 + 3 −6 − 3
2 1 →At x = ; f ″(x) = 2 3 > 0 and at x = ,
(A) (B) 3 3
3 3 3 3
f ″(x) = − 2 3 < 0
1+ 2 3 1+ 3 −6 − 3
(C) (D) \  f (x) has a maximum at x = and a minimum
3 3 3 3 3
 −3 0 −2  −6 + 3
at x =
Solution:  Let the given matrix be A =  1 −1 0  . 3
  2 −6 − 3
 0 a −2  The maximum value of f(x) = y2 = f(x) = at x =
3 3 3
GATE 2015 Solved Paper Set − 1  |  11

The minimum value of f(x) = y1 = f(x) at x Question Number: 38 Question Type: NAT
The signum function is given by
−6 − 3 −2
= = x
3 3 3  ;x≠ 0
sgn ( x ) =  x
From (4)  0; x = 0

−2 2
≤ f(x) ≤ The Fourier series expansion of sgn(cos(t)) has
3 3 3 3
(A) only sine terms with all harmonics.
(B) only cosine terms with all harmonics.
−2 2
⇒  ≤ − 2a ≤ (from (3)) (C) only sine terms with even numbered harmonics.
3 3 3 3 (D) only cosine terms with odd numbered harmonics.
1 −1 −1 1 Solution:
⇒  ≥a≥   ⇒  ≤a≤
3 3 3 3 3 3 3 3  x
 ;x ≠ 0
Given sgn (x) =  | x |
\  The maximum value of ‘a’ such that the matrix A has 0; x = 0

1
three real linearly independent eigen vectors is  cos(t ) π
3 3
 | cos t | ; t ≠ 2
Hence, the correct option is (B). \  sgn (cos (t)) = 
Question Number: 37 Question Type: NAT 0; t = π
 2
A solution of the ordinary differential equation
Its wave form is
d2 y dy 1 − 3e
2
+ 5 + 6 y = 0 is such that y(0) = 2 and y(1) = - 3 . f (t )
dt dt e
1
dy
The value of (0) is ____.
dt p/2
0 2p t
Solution:  Given differential equation is
−1
d2 y dy
2
+5 + 6y = 0(1)
dt dt
The function is half wave symmetric. So its Fourier series
1 − 3e
Where y(0) = 2 and y(1) = − (2) consists of only cosine terms with odd numbered harmonics.
e3
Hence, the correct option is (D).
The auxiliary equation of (1) is
Question Number: 39 Question Type: NAT
D2 + 5D + 6 = 0 Two players, A and B, alternately keep rolling a fair dice.
⇒  D = −2, − 3 The person to get a six first wins the game. Given that player
The general solution of (1) is A starts the game, the probability that A wins the game is
y = c1 e-2t + c2 e-3t (A) 5/11 (B) ½
From (2), y(0) = 2  ⇒  c1 + c2 = 2 (3) (C) 7/13 (D) 6/11
1 − 3e Solution:  Let A and B denote the events of player A getting
And y(1) = − six on the die and player B getting six on the die respectively
e3
1 1 5 5
⇒  c1e-2 + c2e-3 = 3e-2 − e-3(4) \  P(A) = , P(B) = , P ( A) = and P ( B ) =
6 6 6 6
Solving (3) and (4)
Probability of A winning the game
⇒  c1 = 3 and c2 = −1
\  The solution of given differential equation is = P(A) + P ( A ∩ B ∩ A) + P ( A ∩ B ∩ A ∩ B ∩ A) + …
y = 3e−2t − e−3t
=P (A) + P ( A) P ( B ) P(A) + P ( A) P ( B ) P ( A) P ( B )
dy P(A) + … ∞
⇒  = − 6e−2t + 3e−3t
dt 1 5 5 1 5 5 5 5 1 …
= + × × + × × × × +
6 6 6 6 6 6 6 6 6
dy dy
\  (0) = at t = 0 = −6 + 3 = −3. 2 4
dt dt 1 5 1 5 1
= +   × +   × +…∞
Hence, the correct answer is (−3). 6 6 6 6 6
12 | GATE 2015 Solved Paper Set − 1

2
1 5 25 dVo
[which is a geometric series with a = and r =   = ] For the maximum Vo w.r.t P = =0
6 6 36 dP
a
∴  S∞ =
1− r d   1+ x 1 
E  −  =0
1 dP  1 + x + P 1 + P  
= 6
 25  1 1+ x
1−   =
  (1 + P ) (1 + x + P ) 2
2
 36 
6 1 1+ x
= . =
11 1+ P 1+ x + P
6 1 + x + P = (1 + P) 1+ x
∴  Hence probability of A winning the game =
11
Hence, the correct option is (D). 1 + x + P = 1+ x + P ( 1+ x )
Question Number: 40 Question Type: NAT P (1 − 1 + x ) = 1 + x (1 − 1 + x )
P = 1+ x

PR R(1 + X ) Hence, the correct option is (A).


Question Number: 41 Question Type: NAT
+ The circuit shown is meant to supply a resistive load RL from
V0 two separate DC voltage sources. The switches S1 and S2 are
− controlled so that only one of them is ON at any instant. S1
is turned on for 0.2 ms and S2 is turned on for 0.3 ms in a
PR R
0.5 ms switching cycle time period. Assuming continuous
conduction of the inductor current and negligible ripple on
the capacitor voltage, the output voltage Vo (in Volt) across
RL is ______.
S1 L
E S2 +
An unbalanced DC Wheatstone bridge is shown in the figure. 10 V +

+
− 5V C RL V O
At what value of p will the magnitude of V0 be maximum? −

(A) (1+ x ) (B) (1 + x)


10 × 0.2 + 5 × 0.3
Solution:  The average output voltage Vo =
(C) (1+ x ) (D)
(1− x ) 0.5
=7V
Solution:  Given
Hence, the correct answer is 7.
Question Number: 42 Question Type: NAT
PR R(1 + X ) A self commutating switch SW, operated at duty cycle δ is
+
used to control the load voltage as shown in the figure
V0
− VL
D
PR R L
Vdc d C RL
SW VC

+ −
E Under steady state operating conditions, the average voltage
across the inductor and the capacitor respectively, are
ER (1 + x ) R
V= −E 1
PR + R (1 + x ) PR + R VL = 0 and VC =
(A) Vdc
1− δ
 1+ x 1 
=E −  δ 1
 P + 1 + x 1 + P VL = Vdc and VC =
(B) Vdc
2 1− δ
GATE 2015 Solved Paper Set − 1  |  13

δ The region of convergence of its Z-transform would be


VL = 0 and VC =
(C) Vdc
1− δ (A) The region inside the circle of radius 0.5 and cen-
tered at origin.
δ δ
VL = Vdc and VC =
(D) V (B) The region outside the circle of radius 0.25 and
2 1 − δ dc centered at origin.
(C) The annular region between the two circles, both
Solution:  The circuit represents step up chopper. So, centered origin and having radii 0.25 and 0.5.
1 (D) The entire Z plane.
Vc = V Hence the average output voltage of inductor
1− δ dc Solution:  Given x(n) = (−0.25)n u(n) + (0.5)n u(-n − 1)
is zero.
(−0.25)n u(n) is similar to (−1)n u(n)
Hence, the correct option is (A).
So ROC is |Z| > |−0.25| and for (0.5)n
Question Number: 43 Question Type: NAT
The single-phase full-bridge voltage source inverter (VSI), u(− n − 1) The ROC is |Z| < |0.5|
shown in figure, has an output frequency of 50 Hz. It uses ∴  ROC is 0.25 < |Z| < 0.5
unipolar pulse width modulation with switching frequency Hence, the correct option is (C).
of 50 kHz and modulation index of 0.7. For Vin = 100 V
DC, L = 9.55 mH, C = 63.66 µF, and R = 5 Ω, the amplitude Question Number: 46 Question Type: NAT
of the fundamental component in the output voltage Vo (in A parallel plate capacitor is partially filled with glass of
Volts) under steady − state is ___________. dielectric constant 4.0 as shown below. The dielectric strengths
of air and glass are 30 kV/cm and 300 kV/cm, respectively. The
2 2Vs maximum voltage (in kilovolts), which can be applied across
Solution:  Vo = modulation index ×
p the capacitor without any breakdown, is _____.
2 2 ×100
= 0.7 × = 63.05 V
3.14 Air, er = 1.0 5 mm
Hence, the correct answer is 63.05. 10 mm
Question Number: 44 Question Type: NAT Glass, er = 4.0
A 3-phase 50 Hz square wave (6-step) VSI
L
+
+ ε r1 ε o A ε o A
+ Full-bridge
Solution:  C1 = =
VR R VO d d
V in − VSI C
− −
ε r2 ε o A 4ε o A
C2 = =
feeds a 3-phase, 4 pole induction motor. The VSI line voltage d d
has a dominant 5th harmonic component. If the operating C1 C2 4 Aε o
slip of the motor with respect to fundamental component Ceq = =
C1 + C2 5d
voltage is 0.04, the slip of the motor with respect to 5th
harmonic component of voltage is ______. V
Electric field E =
120 × 50 d
Solution:  Synchronous speed Ns = =1500 rpm
4
Dn ρ s Q CV Ceq ⋅V
N ⇒ E= = = = =
5th harmonic speed = s = 300 rpm ε 0 ε 0 A × ε 0 Aε 0 Aε 0
5
Ns 4 × A× ε 0 V
Ns + 30 × 105 = ×
5 5 × 5 ×10 −3
Aε 0
Slip at 5th harmonic =
Ns
75
⇒ V= ×103 =18.75 kV
4
1500 + 300
  = = 1.2 Hence, the correct answer is 17 to 20.
1500
Question Number: 47 Question Type: NAT
Hence, the correct answer is 1.2. The figure shows a digital circuit constructed using negative
Question Number: 45 Question Type: NAT edge triggered J-K flip flops. Assume a starting state of
Consider a discrete time signal given by Q2Q1Q0 = 000. This state Q2Q1Q0 = 000 will repeat after
x[n] = (-0.25)n u[n] + (0.5)n u[-n − 1] ______ number of cycles of the clock CLK.
14 | GATE 2015 Solved Paper Set − 1

D ( A + B)( A + C)
1 J0 Q0 J1 Q1 J2 Q2

Clk Clock Clock Clock


CD
K0 Q0 1 K1 Q1 1 K2 Q2 AB
1 00 01 11 10

00 1
Solution:  By observing the circuit, it is Asynchronous, as
well as synchronous circuits. ACD
01 1
The output of Q0 is connected as Clk pulse to Q1, Q0 flip
flops synchronously.
11
For Q0 flipflop, J0 = K0 = 1, so it works as toggle switch Qn+1
= Qn , for every Clk pulse it will change its state.
10 1 1
Q1 Q2 work as synchronous circuit with Q0 as Clk pulse A BD
negative edge Clk (1 → 0)
Clk Q2  Q1  Q0 J2(Q1)  K2  J1 (Q2)  K1  J0  K0 AC D + ABD
0 0  0   0  0  1   1    1  1  1
Hence, the correct option is (C).
1 0  0   1  0  1   1    1  1  1
Question Number: 49 Question Type: NAT
2 0   1   0  1  1   1    1  1  1
The op-amp shown in the figure has finite gain A = 1000
3 0   1  1  1  1   1    1  1  1 and an infinite input resistance. A step-voltage Vi = 1 mV is
4 1  0   0  0  1   0    1  1  1 applied at the input at time t = 0 as shown. Assuming that
5 1   0  1  0  1   0    1  1  1 the operational amplifier is not saturated, the time constant
6 0   0   0  0  1   1    1  1  1 (in millisecond) of the output voltage Vo is
7 0   0   1  0  1   1    1  1  1
C
Hence, the correct answer is 6.
Question Number: 48 Question Type: NAT 1 μF
R
f(A, B, C, D) = ∏M(0, 1, 3, 4, 5, 7, 9, 11, 12, 13, 14, 150 is −
a maxterm representation of a Boolean function f(A, B, C, 1 kΩ A = 1000
Vi +
D) where A is the MSB and D is the LSB. The equivalent 1 mV + +
minimized representation of this function is −
Vo
(A) (A + C + D)( A + B + D) t=0s −

(B) ACD + ABD

(C) AC D + ABC D + ABC D


(A) 1001 (B) 101
(D) (B + C + D)(A + B + C + D)( A + B + C + D) (C) 11 (D) 1
Solution:  F(A, B, C, D) = πM(0, 1, 3, 4, 5, 7, 9, 11, 12, 13,
Solution:
14, 15)
CD C=1
AB
00 01 11 10
R
00 0 0 0 −
1 kΩ V
+ +
+ V
01 0 0 0 − Vi Vo

11 0 0 0 0

10 0 0
V2 = 0
Vo A(V2 − V1)
Vo = −1000 V1
GATE 2015 Solved Paper Set − 1  |  15

−Vo along with the generation limits for the two units are given
V1 = below:
1000
IR = IC C1(P1) = 0.01 P12 + 30P1 + 10; 100 MW ≤ P1 ≤ 150 MW

 Vs −V1  d C2(P2) = 0.05 P22 + 10P2 + 10; 100 MW ≤ P2 ≤ 180 mW


 R  = C . dt (V1 −V0 ) The incremental cost (in Rs/MWh) of the power plant when
 
it supplies 200 MW is _______.
 V0 
 Vs + 1000  d  −V  Solution:  Given
  = C  0 −V0  G(P1) = 0.01 P12 + 30P1 + 10; 100 MW ≤ P1 ≤ 150 Mw
 1000  dt  1000 
  C2(P2) = 0.05 P22 + 10P2 + 10; 10 MW ≤ P2 ≤ 180 Mw
10-3 Vs + 10-6 Vo P1 + P2 = 200 MW (1)

10 −6 d dC1 dC2
=- [1001 Vo ] =
dP1 dP2
1000 dt
10-3 × 10-3 + 10-6 Vo 0.02P1 + 30 = 0.1P2 + 10
0.1P2 − 0.02P1 = 20 (2)
dVo By solving (1) and (2) P2 = 200; P1 = 0
= −1.001 × 10-6
dt But P2 is more than maximum limit therefore the load
dVo distribution between the units is P1 = 100 Mw and P2 = 100
1 + Vo = −1.001 dC2
dt MW ⇒  = 20 Rs/MWh
dP2
dVo Hence, the correct answer is 20.
⇒ 1.001 + Vo = − 1
dt Question Number: 52 Question Type: NAT
−1 Determine the correctness or otherwise of the following
⇒  Vo(S) = Assertion [a] and the Reason [r].
1.001S + 1
Assertion: Fast decouples load flow method gives
Time constant τ = 1.001 sec. approximate load flow solution because it uses several
T = 1001 msec. assumptions.
Hence, the correct option is (A). Reason: Accuracy depends on the power mismatch vector
Question Number: 50 Question Type: NAT tolerance.
An 8-bit, unipolar Successive Approximation Register type (A) Both [a] and [r] are true and [r] is the correct rea-
ADC is used to convert 3.5 V to digital equivalent output. son for [a].
The reference voltage is + 5 V. The output of the ADC, at the (B) Both [a] and [r] are true but [r] is not the correct
end of 3rd clock pulse after the start of conversion, is reason for [a].
(A) 1010 0000 (C) Both [a] and [r] are false.
(B) 1000 0000 (D) [a] is false and [r] is true.
(C) 0000 0001
(D) 0000 0011 Solution:
Hence, the correct option is (D).
Solution:  8-bit SAR type ADC, reference voltage = +5V
Question Number: 53 Question Type: NAT
5
k = resolution = 8 = 0.0195 A 50 Hz generating unit has H-constant of 2 MJ/MVA. The
2 machine is initially operating in steady state at synchronous
input = 3.5 V speed, and producing 1 pu of real power. The initial value of
11000000 10110000 the rotor angle δ is 5°, when a bolted three phase to ground
10000000 Vo = k × 192 10100000 Vo = k × 176 short circuit fault occurs at the terminal of the generator.
Vo = k × 128 = 3.75 Vo = k × 160 = 3.4375 Assuming the input mechanical power to remain at 1 pu, the
= 2.5 = 3.125 value of δ in degrees, 0.02 second after the fault is _____.
Hence, the correct option is (A). Solution:  Given H = 2 MJ/MVA
Question Number: 51 Question Type: NAT P = 1 p.u.
Consider the economic dispatch problem for a power plant δo = 5°
having two generating units. The fuel costs in Rs/MWh tcr = 0.02
16 | GATE 2015 Solved Paper Set − 1

Pm = 1 p.u. 8
 Idc = =8 A
1
2H (δ cr − δ o )
tcr = 12
π f Pm IAC max = = 8.485 A
12 + 12
2 × 2(δ cr − 50°)
0.02 =  I AC m 
2

π × 50 × 1 Irms = I dc2 +  
δcr = 5.90°  2 
Hence, the correct answer is 5.90°.
8.4852
 = 8 +
2
Question Number: 54 Question Type: NAT = 9.99 A
2
A sustained three-phase fault occurs in the power system
shown in the figure. The current and voltage phasors Hence, the correct answer is 9.99 A.
during the fault (on a common reference), after the natural Question Number: 56 Question Type: NAT
transients have died down, are also shown. Where is the In a linear two-port network, when 10 V is applied to Port
fault located? 1, a current of 4 A flows through Port 2 when it is short-
circuited. When 5 V is applied to Port 1, a current of 1.25
I 1 Transmission Line I3 A flows through a 1 Ω resistance connected across Port
V1 V2
Q S 2. When 3 V is applied to Port 1, the current (in Ampere)
∼ ∼ through a 2 Ω resistance connected across Port 2 is ______.
P Transmission Line
R
I2 I4 Solution:  Consider Y − parameter two port network
I1 = y11 V1 + y22 V2
V2 I2 = y21 V1 + y22 V2
V1 When port − 2 short circuited V2 = 0
I3
I2 I1 I2 4
y21 = = = 0.4
I4 V1 10

(A) Location P In second case: 1.25 = 0.4 × 5 + 1.25 y22


(B) Location Q y22 = −0.6
(C) Location R ⇒  at 3V supply I2 = 3 × 0.4 − 0.6(2I2)
(D) Location S ⇒  I2 = 0.545 A
Hence, the correct answer is 0.5454.
Solution:  The Voltage magnitude of BUS 1 is small there-
Question Number: 57 Question Type: NAT
fore the fault is at Q (or) R. The direction of I2 and I4 are
In the given circuit, the parameter k is positive, and the
quite opposite then there is not fault at R.
power dissipated in the 2 Ω resistor is 12.5 W. The value of
Hence, the correct option is (B).
k is ______
Question Number: 55 Question Type: NAT
The circuit shown in the figure has two sources connected 2Ω 5Ω
in series. The instantaneous voltage of the AC source (in
+ Vo −
Volts) is given by v(t) = 12 sint. If the circuit is in steady
+ 10 Ω
state, then the rms value of the current (in Ampere) flowing

in the circuit is _______. 4V 5A
kVo

V(t ) ∼ 1Ω
Solution:  Given

1H 2Ω 5Ω
+
8V + Vo −
10 Ω
4V 5A
kVo
Solution:  Given V = 12 sint
V=8V
GATE 2015 Solved Paper Set − 1  |  17

V2 and rotational losses are 600 W. The motor operates from a


Power dissipation in 2 Ω Resistor =12.5 W 230 V DC supply. If the motor runs at 1000 rpm, the output
R
torque produced (in Nm) is ________.
⇒ V02 =12.5 × 2 Solution:  Given flux/pole f = 0.01 ωb
⇒  Vo = 5 V P = 4
Z = 666
Vo Parallel path A = 2
Current in 2 Ω resistor = = 2.5 A
2 Armature Resistance Ra = 0.267 Ω
Rotational losses = 600 w
KCL − nodal analysis 2.5 + KVo = 5
Supply voltage V = 230 V
2.5 Speed N = 1000 rpm
=
K = 0.5
5 ϕ ZN P
Eb = ×
Hence, the correct answer is 0.5. 60 A
Question Number: 58 Question Type: NAT 0.01× 666 ×1000 4
A separately excited DC motor runs at 1000 rpm on no load Eb = ×
60 2
when its armature terminals are connected to a 200 V DC
source and the rated voltage is applied to the field winding. Eb = 222 V
The armature resistance of this motor is 1 Ω. The no-load V − Eb 230 − 222
armature current is negligible. With the motor developing Ia = = = 29.96 A
Ra 0.267
its full load torque, the armature voltage is set so that the
rotor speed is 500 rpm. When the load torque is reduce to Output power = Eb Ia − 600
50% of the full load value under the same armature voltage =222 × 29.96 − 600 = 6051.12 W
conditions, the speed rises to 520 rpm. Neglecting the
Pout 6051.12
rotational losses, the full load armature current (in Ampere) Output Torque = = = 57.78 N − m
is ______. W 2π × 1000
60
Solution:  Given N1 = 1000 rpm
Hence, the correct answer is 57.78.
V = 200 V
Ra = 1 Ω Question Number: 60 Question Type: NAT
No load back e.m.f Eb = 200 V A 200/400 V, 50 Hz, two-winding transformer is rated at 20
1
kVA. Its windings are connected as an auto transformer of
N2 = 500 rpm rating 200/600 V. A resistive load of 12 Ω is connected to
T3 = 0.5 T2 the high voltage (600 V) side of the auto-transformer. The
N3 = 520 rpm value of equivalent load resistance (in Ohm) as seen from
Eb1 1000 low voltage side is ______.
=   ⇒  Eb = 100 V
Eb2 500 2 Solution:
600
 100 = V − Ia (1) Transformation ratio K = =3
2 200
Eb1 1000 Resistive load = 12 Ω
=   ⇒  Eb = 104
Eb3 520 3 12
Load resistance on primary side = = 1.33 Ω
9
 104 = V − Ia (2)
3 Hence, the correct answer is 1.33 Ω.
T2 I a2 T2 Ia I a2 Question Number: 61 Question Type: NAT
∝ ⇒ = 2 ⇒ I a3 = (3)
T3 I a3 0.5T2 I a3 2 Two single-phase transformers T1 and T2 each rated at
500 kVA are operated in parallel. Percentage impedances
Solving (1), (2) and (3) Ia = 8 A of T1 and T2 are (1 + j6) and (0.8 + j4.8), respectively. To
2
share a load of 1000 kVA at 0.8 lagging power factor, the
Hence, the correct answer is 8.
contribution of T2(in kVA) is _____.
Question Number: 59 Question Type: NAT
A DC motor has the following specifications: 10 hp, 37.5 A, Solution:  Given Z1 = 1 + j6
230 V; flux/pole = 0.01 Wb, number of poles = 4, number of Z2 = 0.8 + j4.8
conductors = 666, number of parallel paths = 2. Armature SL = 1000 KVA
resistance = 0.267 Ω. The armature reaction is negligible cos∅ = 0.8
18 | GATE 2015 Solved Paper Set − 1

Z1 6.08 Solution:  The second order real system for flat magnitude is
ST = S × = 1000 × = 555.75 KVA
2
Z1 + Z 2 10.94 jx

Hence, the correct answer is 555.75 KVA. °(−2 + j3) *(2 + j3)

Question Number: 62 Question Type: NAT R


In the signal flow diagram given in the figure, u1 and u2 °(−2 − j3) *(2 − j3)
are possible inputs whereas y1 and y2 are possible outputs.
When would the SISO system derived from this diagram be
controllable and observable? Hence, the correct option is (D).
(A) When u1 is the only input and y1 is the only output.
Question Number: 64 Question Type: NAT
(B) When u2 is the only input and y1 is the only output.
(C) When u1 is the only input and y2 is the only output. Y ( s)
Find the transfer function of the system given below.
(D) When u2 is the only input and y2 is the only output. X ( s)
Solution:  From the given block diagram 5

  •

 X 1  =  5 −2   X 1  + 1 u + 0  u u1 y1
 •   2 1   X 2  1 1 1  2
x1
1/s
 X 2  1 1
−2
 y1  1 0   X1 
y =  X  1
 2  1 −1  2 1

2
By Considering U2 is input and y1 is output. The state u2
x2
equations becomes 1/s
1 −1 y2
 • 
 X 1  =  5 −2   X 1  + 0  u 1
 •   2 −1  X 2  1  2
 X 2  +
G1

X  +
y1 = [1 0]  1 
X (s) H Y(s)
X2  Y(s)
+

+
 5 −2  0   −2  G2
 AB =    = 
 2 −1 1   1 
0 −2  G1 G2
Qc = [ B AB ] =   (A) +
1 1  1 − HG1 1 − HG2
det (QC) = 2 (  controllable) G G2
(B) 1 +
 5 −2  1 + HG1 1 + HG2
CA = [1 0]   = [5 − 2]
2 1  G + G2
(C) 1
C  1 0  1 + H ( G1 + G2 )
Qo =   =  
 A 5 −2 
G + G2
det (Q0) = −2 (observable) (D) 1
1 − H ( G1 + G2 )
Hence, the correct option is (B).
Question Number: 63 Question Type: NAT Solution:  The given block diagram can be analyzed by sig-
The transfer function of a second order real system with nal flow graph two forward paths of magnitude = G1, G2
a perfectly flat magnitude response of unity has a pole at Two loops of magnitude = −G1 H, -G2H
(2 − j3). List all the poles and zeroes. By masons gain formula
(A) Poles at (2 ± j3), no zeroes. G1 + G2
(B) Poles at (±2 − j3), one zero at origin. T(s) =
1 + G1 H + G2 H
(C) Poles at (2 − j3), (-2 + j3), zeroes at (−2 − j3), (2 + j3)
(D) Poles at (2 ± j3), zeroes at (-2 ± j3) Hence, the correct option is (C).
GATE 2015 SolvEd PAPEr Set − 1 | 19

Question Number: 65 Question Type: NAT Im


The open loop poles of a third order unity feedback system
are at 0, −1, −2. Let the frequency corresponding to the
point where the root locus of the system transits to unstable
region be K. Now suppose we introduce a zero in the open
loop transfer function at −3, while keeping all the earlier −2 −1 Re
open loop poles intact. Which one of the following is TRUE
about the point where the root locus of the modified system
transits to unstable region?
(A) It corresponds to a frequency greater than K
(B) It corresponds to a frequency less than K Im
(C) It corresponds to a frequency K
(D) Root locus of modified system never transits to
unstable region.
Solution:
−3 −2 −1 Re
K
Given open loop transfer function T1 =
S ( S + 1)( S + 2)
The root locus plot is like
By adding zero the transfer function and root locus plots
So in second case the root locus plot never goes to Right
K ( S + 3) hand side.
T2 =
S ( S + 1)( S + 2) Hence, the correct option is (D).
GATE 2015 Solved Paper
Graduate Aptitude Test in Engineering
Set − 2
Questions Paper Name: EE: ELECTRICAL ENGINEERING 7th Feb Shift1
Number of Questions: 65 Total Marks: 100.0
Wrong answer for MCQ will result in negative marks, (−1/3) for 1 Mark Questions and (−2/3) for 2 Marks Questions.

General Aptitude
Number of Questions: 10 Selection Marks: 15.0
Q.1 to Q.5 carry 1 mark each and Q.6 to Q.10 carry 2 Statements:
marks each. (I)  One fourth of the weight of a pole is 5 Kg.
Question Number: 1 Question Type: MCQ (II) The total weight of these poles is 160 kg more than the
We _______ our friend’s birthday and we ______ how to total weight of two poles.
make it up to him. (A) Statement I alone is not sufficient.
(A) completely forgot --- don’t just know (B) Statement II alone is not sufficient.
(B) forgot completely --- don’t just know (C) Either I or II alone is sufficient.
(C) completely forgot --- just don’t know (D) Both statements I and II together are not sufficient.
(D) forgot completely --- just don’t know
Solution:
Solution:  The correct answer is choice (C). Here, ‘com- I: Weight of the pole = 20 kg
pletely’ modifies ‘forgot’ which is to say that an action was Total weight of 10 poles = (20) (10) kg
missed out on. The same rule applies to the second blank as I is sufficient
well. ‘Don’t’ know’ come together, showing a misinforma- II: Total weight of 8 poles = 160 kg
tion and ‘just’ modifies it, showing an extent.
10
Hence, the correct option is (C). Total weight of 10 poles = (160) kg
8
Question Number: 2 Question Type: MCQ II is sufficient
Choose the statement where underlined word is used Hence, the correct option is (C).
correctly.
Question Number: 5 Question Type: MCQ
(A) The industrialist has a personnel jet.
Consider a function f(x) = 1 - |x| on -1 ≤ x ≤ 1. The value of x
(B) I write my experience in my personnel dairy.
at which the function attains a maximum and the maximum
(C) All personnel are being given the day off.
value of the function are:
(D) Being religious is a personnel aspect.
(A) 0, -1 (B) -1, 0
Solution:  The word ‘personnel’ means a group of people (C) 0, 1 (D) -1, 2
who work for a company or an organization. Choice (C)
uses the word correctly. The rest of the choices should use Solution:  f (x) is maximum when |x| is minimum i.e.,
‘Personal’. when |x| is zero i.e., when x is zero.
Maximum value of f(x) = 1 − 0 = 1. Which occurs at x = 0.
Hence, the correct option is (C).
Hence, the correct option is (C).
Question Number: 3 Question Type: MCQ
A generic term that includes various items of clothing such Question Number: 6 Question Type: MCQ
as a skirt, a pair of trousers and a shirt is Out of the following four sentences, select the most suitable
(A) fabric (B) textile sentence with respect to grammar and usage:
(C) fibre (D) apparel (A) Since the report lacked needed information, it was
of no use to them.
Solution:  The correct answer is (D) apparel. (A) refers to (B) The report was useless to them because there were
the type of apparel. (B) refers to the business that makes no needed information in it.
apparels. (C) is again a material used to make an apparel. (C) Since the report did not contain the needed infor-
Hence, the correct option is (D). mation, it was not real useful to them.
(D)  Since the report lacked needed information, it
Question Number: 4 Question Type: MCQ
would not had been useful to them.
Based on the given statements, select the most appropriate
option to solve the given question. What will be the total Solution:  Choice (A) is free of all errors. Though, the arti-
weight of 10 poles each of same weight? cle ‘the’ before ‘needed’ would render it correct. But, of the
GATE 2015 Solved Paper Set − 2  |  21

given choices, (A) is correct. The use of ‘there were’ in (B) fg(h (2,5,7,3),4,6,8) = fg (1,4,6,8) = f (1,4,6,8) × g(1,4,6,8)
is incorrect. ‘Real useful’ in ungrammatical in (C). ‘Not had = max (1, 4, 6, 8) × min (1, 4, 6, 8) = 8 × 1 = 8
been’ is ungrammatical in (D). Hence, the correct answer is 8.
Hence, the correct option is (A).
Question Number: 9 Question Type: MCQ
Question Number: 7 Question Type: MCQ If the list of letters, P, R, S, T U is an arithmetic sequence,
P which of the following are also in arithmetic sequence?
 I. 2P, 2R, 2S, 2T, 2U
r q  II. P–3, R–3, S–3, T–3, U–3
III. P2, R2, S2, T 2, U 2
(A) I only
S (B) I and II
Q R
p
(C) II and III
(D) I and III
In a triangle PQR, PS is the angle bisector of QPR and
Solution:  P, R, S, T, U is an arithmetic sequence
QPS = 60°. What is the length of PS?
\  R − P = S − R = T − S = U − T. Let each of these equal
(q + r) qr values be k.
(A) (B)
qr ( q + r) I:  2 (R − P) = 2 (S − R) = 2 (T – S) = 2(U − T)
= 2k
(q + r )2 \ 2P, 2R, 2S, 2T, 2U is an arithmetic sequence.
(C) ( q 2 + r 2 ) (D)
qr II. R − 3 − (P − 3) = S − 3 − (R − 3) = T − 3 − (S – 3) = U
− 3 − (T − 3) = k.
Solution:  Area of triangle PQR = Area of triangle PQS+ \  P − 3, R − 3, S − 3, T − 3, U − 3 is an arithmetic
Area of triangle PSR sequence.
1 Hence, the correct option is (B).
Area of triangle PQR = (r) (q) sin∠P
2
Question Number: 10 Question Type: MCQ
1 rq Four branches of a company are located at M, N, O and P.
= (r)(q) sin (2∠QPS) = sin 120°
2 2 M is north of N at a distance of 4 km; P is south of O at a
(   PS is the angle bisector of ∠QPR) distance of 2 km; N is southeast of O by 1 km. What is the
distance between M and P in km?
1 1 1 (A) 5.34 (B) 6.74
\ (r q sin 120°) = (r (PS) sin 60°) + (q (PS) sin 60°)
2 2 2 (C) 28.5 (D) 45.49
 3 3 Solution:  Line diagram
r q   = (PS) (r + q)
 2  2
M
rq
PS =
r+q
Hence, the correct option is (B). O
45°
Question Number: 8 Question Type: MCQ 1
If p, q, r, s are distinct integers such that: 1 km
N
45°
f(p, q, r, s) = max(p, q, r, s) 2 km
1 km
g(p, q, r, s) = min(p, q, r, s)
h(p, q, r, s) = remainder of (p × q)/(r × s) if (p × q) > (r × s) P
or remainder of (r × s)/(p × q) if (r × s) > (p × q)
Also a function fgh(p, q, r, s) = f(p, q, r, s) × g(p, q, r, s) × M
h(p, q, r, s)
Also the same operations are valid with two variable
functions of the form f(p, q) 4 + 2 − 1 = 5 km
What is the value of fg(h(2,5,7,3), 4,6,8)?
Solution: P 1 km
 21 
h(2, 5, 7, 3) = remainder of   = 1 (  (r × s) > (p × q) So MP = (5) 2 + (1) 2 = 25 + 1 = 26 = 5.34 Kms.
 10 
22 | GATE 2015 Solved Paper Set − 2

Electrical Engineering
Number of Questions: 55 Section Marks: 85.0
Q.11 to Q.35 carry 1 mark each and Q.36 to Q.65 carry 2 Marks Question Number: 13 Question Type: NAT
each. Match the following.
Question Number: 11 Question Type: NAT P. Stoke’s Theorem 1. ∫∫ D.ds = Q
Given f(z) = g(z) + h(z), where f, g, h are complex valued
functions of a complex variable z. Which one of the
Q. Gauss’s Theorem 2. ∫ f ( z )dz = 0
following statements is TRUE? R. Divergence Theorem 3. ∫∫∫ (∇. A)dv =  ∫∫ A.ds
(A) If f(z) is differentiable at z0, then g(z) and h(z) are
also differentiable at z0. S. Cauchy’s Integral Theorem 4. ∫∫ (∇ × A).ds = ∫ A.dl
(B) If g(z) and h(z) are differentiable at z0, then f(z) is (A) P–2 Q–1 R–1 S–3
also differentiable at z0. (B) P–4 Q–1 R–3 S–2
(C) If f(z) is continuous at z0, then it is differentiable at z0. (C) P–4 Q–3 R–1 S–2
(D) If f(z) is differentiable at z0, then so are its real and (D) P–3 Q–4 R–2 S–1
imaginary parts.
Solution:  Correct matching is
Solution:  We know that every continuous function need P − 4, Q − 1, R − 3, S − 2.
NOT be differentiable
Hence, the correct option is (B).
\  Option (C) is NOT TRUE
Counter Example for option (A):- Question Number: 14 Question Type: NAT
Let g(z) = 2x + i3y and h(z) = 3x + i2y t
The Laplace transform of f(t) = 2 is s-3/2. The Laplace
\  f(z) = g(z) + h(z) p
= (2x + i3y) + (3x + i2y) transform of g(t) = 1 is
= 5x + i5y p
(A) 3s-5/2/2 (B)
s-1/2
= 5(x + iy) (C) s1/2 (D) s3/2
= 5z, where z = x + iy −3
t
It can be easily observed that g(z) and h(z) does not satisfy Solution:  Given f(t) = 2 and L[f(t)] = s 2
cauchy. Riemann equations p
But f(z) is differentiable 1 1 1 = 1
So, option (A) is NOT TRUE ⇒  f ′(t) = 2. . =
2 t p p p
Option (D) is also NOT TRUE \  f ′(t) = g(t)
We know that the sum of two differentiable functions is We know that L[f ′(t)] = s L [f (t)] − f(0)
always differentiable
Hence option (B) is TRUE.  1 
\  L[g(t)] = L   = L[f ′(t)]
Hence, the correct option is (B).  π t 

Question Number: 12 Question Type: NAT  −3  0


= s s 2  − 2
We have a set of 3 linear equations in 3 unknowns. ‘X ≡ Y’   p
means X and Y are equivalent statements and ‘X ≡ Y’ means −1 −1

X and Y are not equivalent statements. = s2 −0= s2 .


P:  There is a unique solution. Hence, the correct option is (B).
Q:  The equations are linearly independent. Question Number: 15 Question Type: NAT
R:  All eigen values of the coefficient matrix are non zero. Match the following.
S:  The determinant of the coefficient matrix is nonzero.
Instrument Type Used for
Which one of the following is TRUE?
P. Permanent magnet moving coil 1. DC only
(A) P≡Q≡R≡S
Q. Moving iron connected through current 2. AC only
(B) P≡R ≡ Q≡S
transformer
(C) P≡Q ≡ R≡S
R. Rectifier 3. AC and DC
(D) P ≡ Q ≡ R ≡ S
S. Electrodynamometer
Solution:  All the four statements P, Q, R and S are (A) P–1 Q–2 R–1 S–3
equivalent. (B) P–1 Q–3 R–1 S–2
Hence, the correct option is (A). (C) P–1 Q–2 R–3 S–3
(D) P–3 Q–1 R–2 S–1
GATE 2015 Solved Paper Set − 2  |  23

Solution:  PMMC meter can show deflection for only dc Question Number: 18 Question Type: NAT
and for ac average torque produced will be zero. Moving In the following circuit, the input voltage Vin is 100 sin(100
iron instrument along with CT can be used only for AC. πt). For 100 πRC = 50, the average voltage across R(in
Rectifier instrument converter AC to DC and can be used Volts) under steady-state is nearest to
for both AC and DC.
Hence, the correct option is (C). C
Vin +
Question Number: 16 Question Type: NAT + −
∼ R
A 3-phase balanced load which has a power factor of 0.707

is connected to a balanced supply. The power consumed C
by the load is 5 kW. The power is measured by the two-
wattmeter method. The readings of the two wattmeters are
(A) 3.94 kW and 1.06 kW
(B) 2.50 kW and 2.50 kW (A) 100 (B) 31.8
(C) 5.00 kW and 0.00 kW (C) 200 (D) 63.6
(D) 2.96 kW and 2.04 kW Solution:  During positive half cycle of the supply under
Solution:  Total power consumed W1 + W2 = 5 kW (1) steady state
Power factor = cosϕ = cos45° = 0.707 −
Vm
p  3 (W1 −W2 )  +
= ϕ = tan-1   +

− +
4  W1 + W2  R VR

5 −
W1 − W2 = (2) Vm
3 +

Solving (1) and (2), we get During negative half cycle of the supply under steady state.
W1 = 3.94 kW and W2 = 1.06 kW

Hence, the correct option is (A). Vm
+
Question Number: 17 Question Type: NAT +
A capacitive voltage divider is used to measure the bus ∼ + R VR

voltage Vbus in a high-voltage 50 Hz AC system as shown −

in the figure. The measurement capacitors C1 and C2 have Vm
+
tolerances of ±10% on their nominal capacitance values. If
the bus voltage Vbus is 100 kV rms, the maximum rms output Average voltage across Resistance |(VR)avg| = 200 V
voltage Vout (in kV), considering the capacitor tolerance, is
______. Hence, the correct option is (C).
Question Number: 19 Question Type: NAT
Two semi-infinite dielectric regions are separated by a plane
boundary at y = 0. The dielectric constants of region 1(y <
C1 1 µF ± 10% 0) and region 2(y > 0) are 2 and 5, respectively. Region 1 has

V bus uniform electric field E = 3aˆ y + 2aˆ z , where aˆ x , aˆ y and aˆ z
are unit vectors along the x, y and z axes, respectively. The
C2 9 µF ± 10% V out electric field in region 2 is
(A) 3aˆ x + 1.6 aˆ y + 2aˆ z (B) 1.2âx + 4ây + 2âz

C1 3aˆ x + 10 aˆ y + 0.8aˆ z
(C) 1.2âx + 4ây + 0.8âz (D)
Solution:  Vout = Vbus ×
C1 + C2 Solution:  The electric field in region 1 is
(1 + 10%) E1 = 3ax + 4ay + 2az
For maximum Vout = 100 × The electric field in region 2 is
(1 + 10%) + (9 − 10%)
2
1.1 E2 = 3ax + (4ay) + 2az
= 100 × = 11.95 kV 4
(1.1 + 8.1) E2 = 3ax + 1.6 ay + 2az
Hence, the correct option is (11.75 to 12.25). Hence, the correct option is (A).
24 | GATE 2015 Solved Paper Set − 2

Question Number: 20 Question Type: NAT for ω = 2π × 20 ω rad/sec


A circular turn of radius 1 m revolves at 60 rpm about its 103 103 103
diameter aligned with the x-axis as shown in the figure. The Z2 = and |Z2| = =
1 + j 0.4π 1 + (0.4π ) 2 1.6
value of µ0 is 4 π × 10-7 in SI unit. If a uniform magnetic
field intensity
→  103 
H = 107 ẑ A/m is applied, then the peak value of the  
1.6  2
induced voltage, Vturn(in Volts), is______. Vo = 2 ×  = ≅ 1.25
1× 103 1.6
Z
H Hence, the correct answer is 1.1 to 1.4.
Question Number: 22 Question Type: NAT
The following circuit, the transistor is in active mode and
X VC = 2 V. To get VC = 4 V, we replace RC with RC′ . Then the
R1
ratio C is_____.
RC

+10 V
Solution:  Magnetic flux density by,
B = µo H = 4 π × 10-7 × 107 â z
= 4π âz RC

Flux, ϕ = BA = 4π × πr2 sinωt RB VC


−dφ
V= = 4π2r2(-cosωt)ω
dt
2p N
|V| = 4π2r2ω = 4π2r2
60
2π × 60
= 4π 2 ×12 × 10 −Vc 10 − 2
  60 Solution: For Vc = 2 V, Rc = = (1)
Ic Ic
= 8π2 = 248.05 V
Hence, the correct answer is 246 to 250. 10 −Vc 10 − 4
For Vc = 4 V, Rc′ = = (2)
Question Number: 21 Question Type: NAT Ic Ic
The operational amplifier shown in the figure is ideal.
The input voltage (in Volt) is Vi = 2sin(2π × 2000t). The ( 2) Rc′ 6
= = = 0.75
amplitude of the output voltage Vo (in Volt) is ________. (1) Rc 8
0.1 µF Hence, the correct answer is 0.74 to 0.76.
Question Number: 23 Question Type: NAT
1 kΩ 1 kΩ Consider the following Sum of Products expression, F.
Vi −
Vo F = ABC + A B C + ABC + ABC + A B C
+
The equivalent Product of Sums expression is
F = (A + B + C)( A + B + C)( A + B + C)
(A)

F = (A + B + C )(A + B + C)( A + B + C )
(B)
Solution:  Output voltage of invertering amplifier
( −Z 2 ) F = ( A + B + C )(A + B + C )(A + B +C)
(C)
Vo = Vi ×
Z1 F = ( A + B + C)(A + B + C )(A + B + C)
(D)
Z1 = 1 × 10 Ω 2
Solution:  K-map for the expression F = ABC + A B C + A
1 BC+ ABC+ABC.
(1× 103 )
(0.1× 10 −6 S ) 103 The equivalent POS form is
Z2 = =
1 1 + j 0.1× 103 ω
103 + −6
(0.1× 10 S ) (
= A+ B +C )( A+ B +C )( A+ B +C )
GATE 2015 Solved Paper Set − 2  |  25

BC Question Number: 25 Question Type: NAT


A 00 01 10 11 When a bipolar junction transistor is operating in the
0 1 1 0 1 saturation mode, which one of the following statements
1 0 1 0 1 is TRUE about the state of its collector-base (CB) and the
base-emitter (BE) junctions?
Hence, the correct option is (A). (A) The CB junction is forward biased and the BE
junction is reverse biased.
Question Number: 24 Question Type: NAT
(B) The CB junction is reverse biased and the BE
The filters F1 and F2 having characteristics as shown in
junction is forward biased.
Figures (a) and (b) are connected as shown in
(C)  Both the CB and BE junctions are forward
Figure (c).
biased.
Vo / Vi F1 (D)  Both the CB and BE junctions are reverse
baised.
Vi Vo
f1 Solution:  BJT operates in saturation mode when both the
f
junctions are in forward biased mode, operates in cut-off
mode when both the junctions are in reverse biased mode
  (a) and active mode when one of the junction in forward biased
mode and other one in reverse biased mode.
Vo /Vi F2 Hence, the correct option is (C).

Vi Vo Question Number: 26 Question Type: NAT


f2 The synchronous generator shown in the figure is
f supplying active power to an infinite bus via two short,
lossless transmission lines, and is initially is steady
  (b)
state. The mechanical power input to the generator and
R /2 the voltage magnitude E are constant. If one line is
+Vsat tripped at time t1 by opening the circuit breakers at the

two ends (although there is no fault), then it is seen that
R
F1 the generator undergoes a stable transient. Which one of
Vi + Vo the following waveforms of the rotor angle δ shows the
R
F2 transient correctly?
−Vsat
  (c)
Synchronous Generator
The cut-off frequencies of F1 and F2 are f1 and f2 respectively. Line 1
Xs
If f1 < f2, the resultant circuit exhibits the characteristics of a ∼ Infinite Bus
(A) Band-pass filter (B) Band-stop filter 10
E δ Line 2
(C) All pass filter (D) High-Q filter
Solution:  Filter F1 is a low pass filter and Filter F2 is high pass
(a) δ
filter and addition of two filters gives Band stop filter (f1 < f2).
Vo /Vi

δ=o
f1 t1 Time
f
(b) δ
Vo /Vi

δ=o
f2 f t1 Time

Hence, the correct option is (B).


26 | GATE 2015 Solved Paper Set − 2

(c) d R
t1
d=o
Time
V L

(d) d V −V
(A) (B)
t1 L R
d=o
Time −RV
(C) 0 (D)
L2
di
Solution:  By applying KVL: V = iR + L
dt

Solution:  For alternator rotor angle is positive, after fault V


By solving above equation: i(t) = (1− e −t τ )
occurring this rotor angle increases. R
Hence, the correct option is (A). Differentially
Question Number: 27 Question Type: NAT above equation, di(t ) = V × 1 e −t τ = V e t /τ
A 3-bus power system network consists of 3 transmission dt R τ L
lines. The bus admittance matrix of the uncompensated
di 2 (t ) −V 1 −t τ
system is and 2nd order differentially, = × e
dt 2 L τ
 − j6 j3 j4 
 j 3 − j 7 j 5  pu. −VR − t /τ
  = e
 j 4 j 5 − j8 L2

If the shunt capacitance of all transmission lines is 50% d 2 i (t ) −VR


= 2
compensated, the imaginary part of the 3rd row 3rd 2
dt t = 0 L
column element (in pu) of the bus admittance matrix after
compensation is Hence, the correct option is (D).
(A) -j7.0 (B) -j8.5
Question Number: 29 Question Type: NAT
(C) -j7.5 (D) -j9.0
The current i(Ampere) in the 2 Ω resistor of the given
Solution: network is ______.
 y11 − y12 − y13  1Ω
YBus =  − y21 y22 − y23  i
 − y31 − y32 y33  + 1Ω
5V − 1Ω 2Ω
y33 = -j8 = y30 + y31 + y32
-j8 = y30 + (-j4) + (-j5)
1Ω
y30 = j1
j1
Compensation of y30 = = j0.5 Solution:  The network can be redrawn as
2
y33(new) = j0.5 − j4 − j5 = -j8.5
1Ω 1Ω
Hence, the correct option is (B). i
+
Question Number: 28 Question Type: NAT 5V − 2Ω
A series RL circuit is excited at t = 0 by closing a switch as 1Ω
1Ω
shown in the figure. Assuming zero initial conditions, the
d 2t
value of 2 at t = 0+ is
dt For balanced bridge i = 0 A
GATE 2015 Solved Paper Set − 2  |  27

Question Number: 30 Question Type: NAT 0.04 Ω j 0.22 Ω j 0.22 Ω


Find the transformer ratios a and b such that the impedance
(Zin) is resistive and equals 2.5 Ω when the network is 0.05 Ω
Vs j 6.28 Ω Vg
excited with a sine wave voltage of angular frequency of s
5000 rad/s.

C = 10 µF L = 1mH
Solution:  Rotor current I2
Vg 210
Zin R = 2.5 Ω = = = 205.09 A
 R2 
2
12
+ 0 . 22 2

 S  + X2
2
1:b 1:b
 
(A) a = 0.5, b = 2.0
(B) a = 2.0, b = 0.5 Gross torque = Tg
(C) a = 1.0, b = 1.0 R2 (1 − S ) ( 0.05)( 0.95)
3 ( 205.09 ) ×
2
(D) a = 4.0, b = 0.5 3I 22
= S = 0.05
1  1  ω  2π × 2850 
Solution:  Zin = -jXc + 2  jX L + a 2 R   60 
b    
−j 1  1  Tg = 401.86 Nm
Z in = −6
+ 2  j (5000)(10 × 10 −6 ) + 2 ( 2.5) 
(5000 × 10 × 10 ) b  a  Hence, the correct answer is 400 to 403.
1  2.5  Question Number: 33 Question Type: NAT
Zin = -j20 + j5 + 2 
b 2  a  A 4-pole, separately excited, wave wound DC machine
For Zin = 2.5 Ω = 2.5 Ω(Pure resistive) with negligible armature resistance is rated for 230 V and 5
kW at a speed of 1200 rpm. If the same armature coils are
j5
-j20 + 2 = 0 (reactance = 0) reconnected to form a lap winding, what is the rated voltage
b (in volts) and power (in kW) respectively at 1200 rpm of the
1 reconnected machine if the field circuit is left unchanged?
b =
2
and b = 0.5
4 (A) 230 and 5 (B) 115 and 5
2.5 (C) 115 and 2.5 (D) 230 and 2.5
Zin = 2.5 Ω = 2 2   ⇒  a2b2 = 1
ab ϕ ZN P
1 Solution:  Bode emf Eb = ×
a= =2 60 A
b
Hence, the correct option is (B). 1
Eb ∝
Question Number: 31 Question Type: NAT A
A shunt-connected DC motor operates at its rated terminal Eb2 A1 Eb2 2( wave winding)
voltage. Its no-load speed is 200 radian/second. At its torque = ⇒ =
of 500 Nm, its speed is 180 radian/second. The motor is Eb1 A2 230 4(lap winding)
used to directly drive a load whose load torque TL depends
on its rotational speed ωr (in radian/second), such that TL Eb2 =115 V
= 2.78 × ωr. Neglecting rotational losses, the steady-state In wave winding P = VI = 5 kW
speed (in radian/second) of the motor, when it drives this Since number of parallel paths are doubled current also
load, is _______. V 
Solution:  At rated torque of 500 Nm with the relation doubles in lap winding P =   ( 2 I ) = 5 kW
2
TL = 2.78 ωr
Hence, the correct option is (B).
TL 500
ωr = = = 179.856 rad sec Question Number: 34 Question Type: NAT
2.78 2.78 An open loop control system results in a response of e-2t(sin
Hence, the correct answer is 177 to 183. 5t + cos 5t) for a unit impulse input. The DC gain of the
Question Number: 32 Question Type: NAT control system is _____.
The figure shows the per-phase equivalent circuit of a two- Solution:  Response c(t) = e-2t (sin5t + cos5t)
pole three-phase induction motor operating at 50 Hz. The For unit impulse, gain G(s)
“air-gap” voltage, Vg across the magnetizing inductance,
is 210 V rms, and the slip, s, is 0.05. The torque (in Nm) C ( s) 5 ( S + 2)
= = +
produced by the motor is ______. R( s) ( S + 2) + 5 ( S + 2) 2 + 52
2 2
28 | GATE 2015 Solved Paper Set − 2

S +7 Question Number: 36 Question Type: NAT


G ( s) = The volume enclosed by the surface f(x, y) = ex over the
( S + 2) 2 + 25
triangle bounded by the lines x = y; x = 0; y = 1 in the xy
S +7 plane is ______.
DC gain Lt G(s) = Lt
s→0 s →0 ( S + 2) 2 + 25 Solution:  The Volume enclosed by the surface f(x, y) = ex
7 7 over the triangle OAB is
= = = 0.241 y
4 + 25 29 y=1
Hence, the correct answer is 0.23 to 0.25. (0 , 1)B A(1 , 1)
Question Number: 35 Question Type: NAT
Nyquist plots of two functions G1(s) and G2(s) are shown
x
in figure. 0
Im Im ∞ x=y
w=∞ ω
G1(s) ω=0

G1(s) Re
w Re
V = ∫ ∫ f(x, y)dxdy
OAB
0    
1 y
(a) Im
=∫ ∫ e dxdy
x

y =0 x =0
w=0

∞→w
1
 y x 
Re = ∫  ∫ e dx dy
y =0  x =0 
y
(b) Im
1 
= ∫ e  dy
x

y =0  x = 0
1
1 Re = ∫ e y − e 0 dy
y =0

1
(c) Im = ∫ (e y − 1)dy
y =0

1
=e y − y  0
Re
=(e1 − 1) − (e0 − 0)
=e − 2

(d) Im =2.718 − 2

ω =0.718.
Hence, the correct answer is 0.70 to 0.76.
ω Re
Question Number: 37 Question Type: NAT
↓ Two coins R and S are tossed. The 4 joint events HRHS,
0
TRTS, HRTS, TRHS have probabilities 0.28, 0.18, 0.30, 0.24,
Solution: respectively, where H represents head and T represents tail.
1 Which one of the following is TRUE?
Nuquist product of G1(s) and G2(s) = ×S=1
S (A) The coin tosses are independent.
Nyquist plot (B) R is fair, S is not.
Im (C) S is fair, R is not.
(D) The coin tosses are dependent.
Solution:
1 Re
When two coins R and S are tassed, given that the
probabilities of the four joint events are P(HRHS) = 0.28,
Hence, the correct option is (B). P(TRTS) = 0.18, P(HRTS) = 0.30 and P(TRHS) = 0.24
GATE 2015 Solved Paper Set − 2  |  29

Clearly, the coin tosses can’t be independent Question Number: 39 Question Type: NAT
For, Let P(HR) = r and P(HS) = s Consider a signal defined by
⇒  P(TR) = 1 − r and P(TS) = 1 − s e j10 t for t ≤ 1
If the coin tosses are independent, then x(t) = 
 0 for t >1
P(HRHS) = P(HR) P(HS) = 0.28
⇒  rs = 0.28 (1) Its Fourier Transform is
P(TRTS) = P(TR) P(TS) = 0.18 2 sin (ω −10 ) sin(ω − 10)
⇒ (1 − r) (1 − s) = 0.18 (2) (A) 2e j10
(B)
P(HRTS) = P(HR) P(TS) = 0.30 ω −10 ω − 10
⇒  r(1 − s) = 0.30 (3) 2 sin ω 2 sin ω
P(TRHS) = P(TR) P(HS) = 0.24 (C) e j10ω
(D)
ω −10 ω
⇒ (1 − r) s = 0.24 (4)
⇒  s − rs = 0.24 Solution:
⇒  s − 0.28 = 0.24
e j10 t for | t |≤ 1
⇒  s = 0.52 Given x(t) = 
From (3), r − rs = 0.30 0 for | t |> 1
⇒  r − 0.28 = 0.30 \  The Fourier Transform of x(t) is
⇒  r = 0.58 ∞ 1

∫ x (t )e ∫e
− jω t
Substituting the values of r and s in (2), we have F[x(t)] = dt = j10 t
e − jωt dt
(1 − 0.58) (1 − 0.52) = 0.18 −∞ −1

⇒  (0.42) (0.48) = 0.18 1


e (10 −ω ) jt 
1
0.2016 = 0.18 which is a absurd.
∫e
(10 −ω ) jt
= dt = 
Hence the coin tosses are dependent. −1 (10 − ω ) j  −1
Hence, the correct option is (D).
e (10 −ω ) j e − (10 −ω ) j
Question Number: 38 Question Type: NAT = −
di (10 − ω ) j (10 − ω ) j
A differential equation - 0.2i = 0 is applicable over -10
dt
< t ± 10. If i(4) = 10, then i(-5) is ______. 2  e (10 −ω ) j − e − (10 −ω ) j 
=
Solution:  Given differential equation is
di
- 0.2i = 0 (1) (10 − ω )  2j


dt
i(4) = 10 (2) −2  e (ω −10 ) j − e (ω −10 ) j 
=  
di (ω − 10)  2j 
From (1), = 0.2i
dt
2  e (ω −10 ) j − e − (ω −10 ) j 
=
(ω − 10)  
1
⇒  di = 0.2dt 2j 
i
Integrating on both sides 2
= sin(w − 10).
1 (ω − 10)
∫ di = ∫0.2dt
i Hence, the correct option is (A).
⇒  lni = 0.2t + c Question Number: 40 Question Type: NAT
⇒  i = e0.2t+c The coils of a wattmeter have resistances 0.01 Ω and 1000
⇒  i = c1e0.2t  Where c1 = ec Ω; their inductances may be neglected. The wattmeter is
Given i(4) = 10 connected as shown in the figure, to measure the power
⇒ 10 = c1e0.2 × 4 consumed by a load, which draws 25 A at power factor 0.8.
⇒  c1e0.8 = 10 The voltage across the load terminals is 30 V. The percentage
⇒  c1 = 10e−0.8 error on the wattmeter reading is ______.
Substituting the value of c1 in (3), we get
i = 10e-0.8 e0.2t
i = 10e0.2t − 0.8
Now i(−5) = 10e0.2 × (−5) − 0.8
=10e-1.8 Load
10
= 1.8 = 1.653.
e
Hence, the correct answer is 1.6 to 1.7.
30 | GATE 2015 Solved Paper Set − 2

Solution: 15 TON = 45 TOFF


2 2
V 30
Wattmeter error due to pressure coil = = = 0.9 TOFF 1 T +T 4
R 1000 = ⇒ OFF ON =
TON 3 TON 3
Wattmeter reading = VI cosq
= 30 × 25 × 0.8 = 600 W TON
Duty ratio, δ = = 0.75
0.9 T
% error = × 100 = 0.15%
600 Hence, the correct answer is 0.75.
Hence, the correct answer is 0.14 to 0.16. Question Number: 43 Question Type: NAT
Question Number: 41 Question Type: NAT In the given rectifier, the delay angle of the thyristor T1
A buck converter feeding a variable resistive load is shown measured from the positive going zero crossing of Vs is
in the figure. The switching frequency of the switch S is 30°. If the input voltage Vs is 100 sin(100πt) V, the average
100 kHz and the duty ratio is 0.6. The output voltage VO voltage across R(in Volt) under steady-state is _____.
is 36 V. Assume that all the components are ideal, and that
the output voltage is ripple-free. The value of R(in Ohm) T1 D3 +
that will make the inductor current (iL) just continuous is Vs
_______. +
∼ − R Vo

S iL 5 mH D4 D2

60 V+ +
36 V Vo R
Solution:  Average output voltage, (Vo)avg
π 2π
1 1
Solution:  The inductor ripple current, ∆I =
Vo (Vs − Vo ) =
2π ∫ Vm sin ωt dωt +

2π ∫
π
− Vm sin ωt dωt
fLVs
Vm V
36(60 − 36) (Vo )avg = [1 + cos α ] − m [cos π − cos 2π ]
∆I = 2π 2π
100 × 10 −3 × 5 × 103 × 60
∆I = 0.0288 A Vm 100
(Vo)avg = (3 + cosa) = [3 + cos30]
DI 2p 2p
The average inductor current, IL = = 0.0144 A
2 (Vo)avg = 61.56 V
Vo 36 Hence, the correct answer is 61 to 62.
IL = =
R R Question Number: 44 Question Type: NAT
R = 2500 Ω For linear time invariant systems, that are Bounded Input
Hence, the correct answer is 2480 to 2520 Bounded Output stable, which one of the following
statements is TRUE?
Question Number: 42 Question Type: NAT (A) The impulse response will be integrable, but may
For the switching converter shown in the following figure, not be absolutely integrable.
assume steady-state operation. Also assume that the (B) The unit impulse response will have finite support.
components are ideal, the inductor current is always positive (C) The unit step response will be absolutely integrable.
and continuous and switching period is Ts. If the voltage VL (D) The unit step response will be bounded.
is as shown, the duty cycle of the switch S is _____.
VL
Solution:  Unit step response will not be absolutely integra-
+ − Vo ble but it will be bounded.
15 V
L
+ + VL Ts t Hence, the correct option is (D).
− S C R
− −45 V Question Number: 45 Question Type: NAT
The z-Transform of a sequence x[n] is given as X(z) = 2z +
4 − 4/z + 3/z2. If y[n] is the first difference of x[n], then Y(z)
is given by
Solution:  For continuous conduction, the average inductor
(A) 2z + 2 − 8/z + 7/z2 − 3/z3
voltage VL = 0
(B) −2z + 2 − 6/z + 1/z2 + 3/z3
T T (C) −2z − 2 + 8/z − 7/z2 + 3/z3
VL = 15 × ON + (-45) OFF = 0
TS TS (D) 4z − 2 − 8/z − 1/z2 + 3/z3
GATE 2015 Solved Paper Set − 2  |  31

Solution:  y(n) is first difference of X(n) = x(n) − x(n − 1) J1(Q0)  K1 (Q0) J0 (Q1)  K0(Q1) Q1   Q0
Y(z) = X(z) − Z-1 x(z) – – – – 0 0
Y(z) = [2z + 4 − 4Z-1 + 3Z-2] − [2 + 4z-1 − 4z-2 − 3z-3]
0 1 1 0 0 1
8 7 3
Y(z) = 2z + 2 - + 2 - 3 1 0 1 0 1 1
z z z
1 0 0 1 1 0
Hence, the correct option is (A). 0 1 0 1 0 0
Question Number: 46 Question Type: NAT
Two semi-infinite conducting sheets are placed at right So, initial output at Q1 Q0 − 0 0, the next output is 01.
angles to each other as shown in the figure. A point charge Hence, the correct option is (B).
of +Q is placed at a distance of d from both sheets. The net Question Number: 48 Question Type: NAT
Q2 K A Boolean function f(A, B, C, D) = ∏(1, 5, 12, 15) is to be
force on the charge is . where K is given by
4π ∈0 d 2 implemented using an 8 × 1 multiplexer (A is MSB). The
y inputs ABC are connected to the select inputs S2 S1 S0 of the
multiplexer respectively.
d
+Q
0
d 1
2
3 f (A, B, C, D)
x
4
5
6
1 1 7 S S S
(A) 0 (B) − i − j 2 1 0
4 4
A B C
1 1 1− 2 2 1− 2 2
− i − j (D)
(C) i+ j Which one of the following options gives the correct inputs
8 8 8 2 8 2
to pins 0, 1,2,3,4,5,6,7 in order?
Solution:  The net force on the charge is F = F1 (A) D, 0, D, 0, 0, 0, D , D
1 Q2  1 
F= 2 
−2dax − 2day + ( 2dax + 2day )  (B) D , 1, D , 1, 1, 1, D, D
4π ∈o ( 2d )  2 2 
(C)
D, 1, D, 1, 1, 1, D , D
1 Q 1 − 2 2
2
1− 2 2  (D)
D , 0, D , 0, 0, 0, D, D
F= 2 
ax + ay 
4π ∈o d  8 2 8 2  Solution:  Boolean function f(A, B, C, D) = π(1, 5, 12, 15)
1 Q2 The min term function f(A, B, C, D) = Σm(0, 2, 3, 4, 6, 7, 8,
F= . (k ) 9, 10, 11, 13, 14)
4π ∈o d 2
D I 0 I1 I2 I3 I4 I5 I6 I7
Hence, the correct option is (D). 0 0 2 4 6 8 10 12 14
Question Number: 47 Question Type: NAT 1 1 3 5 7 9 11 3 15
In the following sequential circuit, the initial state (before
D 1 D 1 1 1 D D
the first clock pulse) of the circuit is Q1 Q0 = 00. The state
(Q1Q0), immediately after the 333rd clock pulse is Hence, the correct option is (B).
Q0 Q1 Question Number: 49 Question Type: NAT
J 0 Q0 J 1 Q1 1 kΩ

K 0 Q0 K 1 Q1
+10 V
Clk 0.25 µF
+ v0

(A) 00 (B) 01 2 kΩ
(C) 10 (D) 11 −10 V

Solution:  Johnson counter (MOD-4), no. of cycles com-


2 kΩ
333
pleted = = 83
4
Remainder clock is 1.
32 | GATE 2015 Solved Paper Set − 2

The saturation voltage of the ideal op-amp shown below is The geometric mean radius (GMR) (in cm) of the composite
±10 V. The output voltage v0 of the following circuit in the conductor is kR. The value of k is ______.
steady-state is 3R
(A) square wave of period 0.55 ms.
(B) triangular wave of period 0.55 ms. R
60°
(C) square wave of period 0.25 ms.
(D) triangular wave of period 0.25 ms.
R2 2
Solution:  β = = = 0.5 60°
R1 + R2 4
 1+ β 
Time period, T = 2 RC log  
 1− β 
 1 + 0.5  Solution:  For symmetrical spacing,
T = 2 × 1 × 103 × 0.25 × 10-6 log  
 1 − 0.5  GMR = GMR1 GMR 2 GMR 3
T = 0.55 m sec GMR = GMR1 = 3
0.7788 R × 3R × 3R
Astable multivibrator producer square wave with time
period 0.55 m sec. GMR = KR = ( 3
0.7788 × 3 × 3 R )
Hence, the correct option is (A). K = 1.913
Question Number: 50 Question Type: NAT Hence, the correct answer is 1.85 to 1.95.
The incremental costs (in Rupees/MWh) of operating two Question Number: 52 Question Type: NAT
generating units are functions of their positive powers P1 A 3-kphase transformer rated for 33 kV/11 kV is connected
and P2 in MW, and are given by in delta/star as shown in figure. The current transformers
dC1 (CTs) on low and high voltage sides have a ratio of 500/5.
= 0.2P1 + 50 Find the currents i1 and i2, if the fault current is 300 A as
dP1
shown in figure.
dC2
= 0.24P2 + 40 a
dP2
b
Where
20 MW ≤ P1 ≤ 150 MW c 300 A
i2
20 MW ≤ P2 ≤ 150 MW. i1
For a certain load demand, P1 and P­2 have been chosen such
that dC1/dP1 = 76 Rs/MWh and dC2/dP2 = 68.8 `/MWh. If
the generations are rescheduled to minimize the total cost, i1 = 1/ 3 A, i2 = 0 A
(A)
then P2 is ______. i1 = 0 A, i2 = 0 A
(B)
Solution:  The incremental fuel cost equations are i1 = 0 A, i2 = 1/ 3 A
(C)
dc1
= 76 = 0.2P1 + 50  ⇒  P1 = 120 i1 = 1 3 A, i2 = 1/ 3 A
(D)
d p1
Solution:  Since fault occurred on secondary side and out-
dC2 side of the CT connected winding, i2 = 0 A
= 68.8 = 0.24 P2 + 40  ⇒  P2 = 130
dP2 Current through secondary of the transforms
and P1 + P2 = 250 (1) 5
IS = 300 × =3A
dC dC 500
For the total cost minimization λ = 1 = 2 Current through Primary of the transformer
dP1 dP2
11
0.2P1 + 50 = 0.24P2 + 40 (2) = IP = 3 × =1A
33
0.2(250 − P2) + 50 = 0.24P2 + 40
I
P2 = 136.36 MW Phase current Iph = p
3
Hence, the correct answer is 135 to 137.
1
Question Number: 51 Question Type: NAT i1 = Iph = A
A composite conductor consists of three conductors of 3
radius R each. The conductors are arranged as shown below. Hence, the correct option is (A).
GATE 2015 SolvEd PAPEr Set − 2 | 33

Question Number: 53 Question Type: NAT (100∠ − 120°) − (100∠120°) (100∠0°) − (100∠120°)
A balanced (positive sequence) three-phase AC voltage i= +
j1 − j1
source is connected to a balanced, star connected load through
a star-delta transformer as shown in the figure. The line-to- i = 173.2 �120°
Ip Is
Hence, the correct option is (B).
a A Question Number: 55 Question Type: NAT
∼ R
A symmetrical square wave of 50% duty cycle has
B amplitude of ±15 V and time period of 0.4π ms. This square
wave is applied across a series RLC circuit with R = 5 Ω,
∼ ∼ R R L = 10 mH, and C = 4 µF. The amplitude of the 5000 rad/s
component of the capacitor voltage (in Volt) is _______.
b c
c Solution:
L C

line voltage rating is 230 V on the star side, and 115 V on +


the delta side. If the magnetizing current is neglected and − R
I s = 100∠0° A, then what is the value of I p in Ampere?
(A) 50∠30° (B) 50∠−30°
(C) 50 3 ∠30° (D) 200 ∠30° Hence, the correct answer is 190 to 192 V.
Solution: In transformer (KVA)primary = (KVA)secondary Question Number: 56 Question Type: NAT
Two identical coils each having inductance L are placed
3 V p I p = 3Vs I s
together on the same core. If an overall inductance of aL is
3 × 230 × I p = 3 ×115 ×100 obtained by interconnecting these two coils, the minimum
Ip = 50 A value of a is _____.
Line current star primary leads line current of ∆ by 30°. Solution: Overall inductance = Leq = L1 + L2 ± 2 M
Hence, the correct option is (A). = L + L ± 2 k LL
Question Number: 54 Question Type: NAT Leq is minimum value when k = 1 and two coils are
In the given network V1 = 100 ∠0° V, V2 = 100 ∠ − 120° opposing.
V, V3 = 100 ∠ + 120° V. The phasor current i (in Ampere) is Leq = L + L − 2L = 0
− j1 Hence, the correct answer is 0.
V1
∼ Question Number: 57 Question Type: NAT
j1 A three-winding transformer is connected to an AC voltage
V2

source as shown in the figure. The number of turns are as
V3 i follows: N1 = 100, N2 = 50, N3 = 50. If the magnetizing

current is neglected, and the currents in two windings are
(A) 173.2 ∠−60° (B) 173.2 ∠120° I 2 = 2 ∠30° A and I 3 = 2∠150° A, then what is the value
(C) 100.0 ∠−60° (D) 100.0 ∠120° of the current I1 in Ampere?
V −V V −V I1 N2
Solution: Applying KCL: i + 3 2 + 3 1 0 .
j1 − j1 I2
V −V V −V
i= 2 3+ 1 3 ∼
j1 −j
I3
V 3(t )
i
N1 N3

V3 ∼ ∼ ∼ V1 (A) 1 ∠90° (B) 1 ∠270°


V2 (C) 4 ∠90° (D) 4 ∠270°
j1
− j1 Solution: Primary current = I1 = I 2 + I 3
50 50
I1 = 2 × ∠30° + 2 × ∠150°
0V 100 100
34 | GATE 2015 Solved Paper Set − 2

I1 = 1∠30° + 1∠150° = 1∠90° Solution:  Slip compounding to maximum rotor torque,


r
 Sm = 2
Hence, the correct option is (A). X2
Question Number: 58 Question Type: NAT 0.08
With an armature voltage of 100 V and rated field winding 0.15 =
X2
voltage, the speed of a separately excited DC motor driving  
a fan is 1000 rpm, and its armature current s 10 A. The  X2 = 0.2 Ω
armature resistance is 1 Ω. The load torque of the fan load For Textra = Temax
is proportional to the square of the rotor speed. Neglecting Textra 2
rotational losses, the value of the armature voltage (in Volt) = =1
Temax 1
which will reduce the rotor speed to 500 rpm is ______. + Sm
Sm
Solution:  Back emf Eb1 = V − R.R = 100 − 10 × 1 = 90 V
r21
Torque, T a N2 hence Io a N2  Sm = =1
2
X2
2
I2  N2   500 
=  ⇒ I 2 =10   = 2.5 A r1 = X 2 = 0.2 Ω
I1  N 1   1000     2
External resistance = 0.2 − 0.03 = 0.17 Ω
Eb2 N2  500  Hence, the correct answer is 0.16 to 0.18.
Back emf Eb a N  ⇒  = ⇒ Eb2 = 90 ×  
Eb1 N1  1000  Question Number: 61 Question Type: NAT
Eb2 = 45 V Two three-phase transformers are realized using single-
phase transformers as shown in the figure.
V = Eb2 + I 2 R = 45 + 2.5 × 1 = 47.5 V
A2 A1 a1 a2
Hence, the correct answer is 47.5 V. V1
B2 B1 b1 b2
Question Number: 59 Question Type: NAT
A three-phase, 11 kV, 50 Hz, 2 pole, stars connected, C2 c2
C1 c1
cylindrical rotor synchronous motor is connected to an 11
kV, 50 Hz source, Its synchronous reactance is 50 Ω per
phase, and its stator resistance is negligible. The motor has A2 A1 a1 a2
a constant field excitation. At a particular load torque, its
B2 b2 V2
stator current is 100 A at unity power factor. If the load B1 b1
torque is increased so that the stator current is 120 A, then
the load angle (in degrees) at this load is ________. C2 C1 c1 c2

Solution:
11
Find excitation Ef = V − Ia × R ×103 − j (100 × 50 ) The phase difference (in degree) between voltages V1 and
3 V2 is ______.
|Ef| = 8082.23 V
Solution:  The transformer secondaries are connected in Y
(IaR)2 = E 2f + V 2 − 2 E f V cos δ and ∆ hence phase angle difference between them is 30°.
(120 × 50)2 Hence, the correct answer is 30.
= (8082.23)2 + (6350)2 − 2(8082.23)(6350)cos
Question Number: 62 Question Type: NAT
δ = -47.270
The following discrete-time equations result from the
Hence, the correct answer is -48 to -46°. numerical integration of the differential equations of an
Question Number: 60 Question Type: NAT un-damped simple harmonic oscillator with state variable
x and y. The integration time step is h.
A 220 V, 3-phase, 4-pole, 50 Hz induction motor of wound
rotor type is supplied at rated voltage and frequency. The xk + 1 − xk
= yk
stator resistance, magnetizing reactance, and core loss are   h
negligible. The maximum torque produced by the rotor is
yk + 1 − yk
225% of full load torque and it occurs at 15% slip. The = − xk
actual rotor resistance is 0.03 Ω/phase. The value of external h
resistance (in Ohm) which must be inserted in a rotor phase For this discrete-time system, which one of the following
if the maximum torque is to occur at start is ________. statements is TRUE?
GATE 2015 Solved Paper Set − 2  |  35

(A) The system is not stable for h > 0 Question Number: 64 Question Type: NAT
1 An open loop transfer function G(s) of a system is
(B) The system is stable for h >
p K
G ( s) =
1 s( s + 1)( s + 2)
(C) The system is stable for 0 < h <
2p For a unity feedback system, the breakaway point of the root
1 1 loci on the real axis occurs at,
(D) The system is stable for <h< (A) -0.42
2p p
Solution: (B) -1.58
(C) -0.42 and -1.58
Hence, the correct option is (A). (D) none of the above
Question Number: 63 Question Type: NAT
The unit step response of a system with the transfer function Solution:  Characteristic equation 1 + G(s) H(s) = 0
S(S + 1)(S + 2) = K = 0
1 − 2s
G(s) = is given by which one of the following S[S2 + 3S + 2] + K = 0
1+ s -K = S3 + 3S2 + 2S
waveforms?
dk
(a) y − = 3S2 + 6S + 2 = 0
1 ds
S = -0.42 and -1.58
0
5 t For k > 0, S = -0.42
Hence, the correct option is (A).
−2 Question Number: 65 Question Type: NAT
y For the system governed by the set of equations:
(b) dx1/dt = 2x1 + x2 + u
2
dx2/dt = -2x1 + u
1 y = 3x1
the transfer function Y(s)/U(s) is given by
0
5 t (A) 3(s + 1)/(s2 − 2s + 2)
(c) y (B) 3(2s + 1)/(s2 − 2s + 1)
2 (C) (s + 1)/(s2 - 2s + 1)
1 (D) 3(2s + 1)/(s2 − 2s + 2)
0 Solution:
5 t
−0.75
•
(d) y  x1  =  2 1   x1  + 1 [4]
1  •   −2 0   x2  1
 x2 
0
5 t
x 
−2
[4] = [3 0 ]  1 
 x2 

1 − 2S Y ( s)
Solution:  Transfer function G(s) = Transfer function = C[SI − A]-1 B
1+ S U ( s)
−1
1  1 − 2S    S 0   2 1   1
Y(s) = unit step response =   = [3 0 ]   −   
S  1 + 2S    0 S   −2 0   1
1 −3 S 1  1
= +
S 1+ S = [3 0 ]   
   −2 S − 2  1

y(t) = u(t) − 3e-t u(t) 3( S + 1)


=
y(t) = (1 − 3e-t) u(t) S − 2S + 1
2

Hence, the correct option is (A). Hence, the correct option is (A).
GATE 2016 SOLVED PAPER
ELECTRICAL ENGINEERING
Set – 1
Number of Questions: 55 Total Marks:100.0

Wrong answer for MCQ will result in negative marks, (−1/3) for 1 Mark Questions and (−2/3) for 2 Marks Question.

GENERAL APTITUDE
Number of Questions: 10 Section Marks: 15.0

Q.1 to Q.5 carry 1 mark each and Q.6 to Q.10 carry 2 Solution: Each group contains five consecutive letters from
marks each. the English alphabet. The arrangement within the group is
similar in (A), (B) and (C), but different in (D).
Question Number: 1 Question Type: MCQ
The man who is now Municipal Commissioner worked as. Hence, (D) is the odd one.
(A) the security guard at a university Hence, the correct option is (D)
(B) a security guard at the university Question Number: 5 Question Type: MCQ
(C) a security guard at university
In a quadratic function, the value of the product of the roots
(D) the security guard at the university
(α, β) is 4. Find the value of α + β
n n

Solution: The reference is to a particular person who α−n + β −n


worked as a security guard. Hence the correct option is ‘a (A) n4 (B) 4n
security guard at the university’. (C) 22n–1 (D) 4n–1
Hence, the correct option is (B) Solution: Let the quadratic equation (not function: a function
has zeroes while an equation has roots) be ax2 + bx + c = 0.
Question Number: 2 Question Type: MCQ The roots are α,β
Nobody knows how the Indian cricket team is going to cope b c
with the difficult and seamer-friendly wickets in Australia. ∴α+β= and αβ = . Given αβ = 4
a a
Choose the option which is closest in meaning to the under- α n + β n = α n + β n αn βn = (αβ)n = 4n.
lined phrase in the above sentence.
α−n + β −n αn + β n
(A) put up with (B) put in with
Hence, the correct option is (B)
(C) put down to (D) put up against
Question Number: 6 Question Type: MCQ
Solution: To cope with something or someone is to put up
Among 150 faculty members in an institute, 55 are connected
with or bear with something or someone.
with each other through Facebook® and 85 are connected
Hence, the correct option is (A) through WhatsApp®. 30 faculty members do not have Facebook®
or WhatsApp® accounts. The number of faculty members con-
Question Number: 3 Question Type: MCQ
nected only through Facebook® accounts is _________
Find the odd one in the following group of words. mock, (A) 35 (B) 45
deride, praise, jeer (C) 65 (D) 90
(A) mock (B) deride Solution: The data is shown in the Venn diagram below. F
is Facebook and W is whatsApp
(C) praise (D) jeer
Solution: The words mock, deride, and jeer convey the
same meaning. Praise is the odd man out.
Hence, the correct option is (C)
Question Number: 4 Question Type: MCQ
Pick the odd one from the following options.
(A) CADBE (B) JHKIL
(C) XVYWZ (D) ONPMQ
GATE 2016 Solved Paper Set − 1  |  37

As 30 faculty members have neither account, 120 have Question Number: 9 Question Type: MCQ
accounts. As 55 have a Facebook account and 85 have a In a 2 × 4 rectangle grid shown below, each cell is a rectan-
WhatsApp account, but only 120 have either account 55 + gle. How many rectangles can be observed in the grid?
85 – 120, i.e. 20 members have both accounts. The number
of faculty members who have only a Facebook account is
55 – 20, i.e. 35.
Hence, the correct option is (A)
Question Number: 7 Question Type: MCQ
Computers were invented for performing only high-end use- (A) 21 (B) 27
ful computations. However, it is no understatement that they (C) 30 (D) 36
have taken over our world today. The internet, for exam-
Solution:  To select a rectangle from the grid, From the 5
ple, is ubiquitous. Many believe that the internet itself is an
vertical lines we have to select 2 and from the 3 horizontal
unintended consequence of the original invention. With the
lines we have to select 2. This can be done in 5C2 3C2 viz
advent of mobile computing on our phones, a whole new
10(3) or 30 ways.
dimension is now enabled. One is left wondering if all these
developments are good or, more importantly, required. Hence, the correct option is (C).
Which of the statement (s) below is/are logically valid and Question Number: 10 Question Type: MCQ
can be inferred from the above paragraph?
(i) The author believes that computers are not good
for us.
(ii) Mobile computers and the internet are both in-
tended inventions
(A) (i) only (B) (ii) only
(C) both (i) and (ii) (D) neither (i) nor (ii)
Solution:  The author is expressing doubt whether comput-
ers are good. He/She is not concluding that the computers
are not good. Hence, (i) is not valid.
According to the author many people think internet is an
unintended consequence, but did not make such comment
about mobile computers. Hence, (ii) is not valid.
Hence, neither (i) nor (ii) is valid.
Hence, the correct option is (D). Choose the correct expression for f(x) given in the graph.
(A) f(x) = 1 – |x – 1| (B) f(x) = 1 + |x + 1|
Question Number: 8 Question Type: MCQ
(C) f(x) = 2 – |x – 1| (D) f(x) = 2 + |x – 1|
All hills-stations have a lake. Ooty has two lakes.
(i) Ooty is not a hill-station. Solution:  The graph shows a V which opens downwards.
(ii) No hill-station can have more than one lake. The mod expression has to be preceded by a negative sign
(A) (i) only (B) (ii) only (we reject B, D) at the vertex of the V, i.e. at x = 1, y is 2. We
(C) both (i) and (ii) (D) neither (i) nor (ii) accept C and reject A.
Hence, the correct option is (C).
Solution:  The statement ‘All hill-stations have a lake’ does
not mean that the hill stations have ‘only one lake’. Hence,
neither (i) nor (ii) is logically valid.
Hence, the correct option is (D).
38 | GATE 2016 Solved Paper Set − 1

ELECTRICAL ENGINEERING
Number of Questions:55 Section Marks:57

Q.1 to Q.25 carry 1 mark each and Q.26 to Q.55 carry Let g(t) = e2t sin(5t)
2 marks each.
5
Question Number: 1 Question Type: NAT ∴ L  g (t ) = L e 2t sin (5t ) = = g ( s)
( s − 2 )2 + 5 2
The maximum value attained by the function
From the second shifting theorem, we know that
f(x) = x(x – 1) (x – 2) in the interval [1, 2] is _______
L  g (t − a ) u (t − a ) = g ( s ) e − as
Solution:  We have to find the maximum value attained by
the function f(x) = x(x – 1) (x – 2) we known that f(1) = f(2) Here a = 0
= 0 in [1, 2]. and f(x) < 0 for 1 < x < 2
∴ L  f (t ) = L e 2t sin (5t ) .u (t )
∴The maximum value of f(x) in [1, 2] is 0
Hence, the correct answer is (0) = L  g (t ) .u (t )
Question Number: 2 Question Type: NAT
= L  g (t − 0 ) .u (t − 0 )
Consider a 3 × 3 matrix with every element being equal to
1. Its only non-zero eigenvalue is _______.
= g ( s ) e − os
Solution:  The 3 × 3 matrix with every element in it
5
1 1 1 =
being equal to 1 is 1 1 1
( s − 2 )2 + 5 2
 
1 1 1 5
=
1 1 1 s 2 − 4 s + 29
Let A =   Hence, the correct option is (A)
1 1 1
1 1 1
Question Number: 4 Question Type: MCQ
The characteristic equation of A is A function y(t) such that y(0) = 1 and y(1) = 3e-1is a solution
of the differential equation
A − lΙ = 0
d2 y dy
2
+ 2 + y = 0 then y (2) is______.
1−l 1 1 dt dt
⇒ 1 1−l 1 =0 (A) 5e–1 (B) 5e–2
(C) 7e
–1
(D) 7e–2
1 1 1−l
Solution:  Given differential equation is
⇒ −l 3 + 3l 2 = 0 d2 y dy
+ 2 + y = 0             (1)
⇒ l (l − 3) = 0
2
dt 2 dt
Also given y (0) = 1 and y (1) = 3e–1     (2)
⇒ l = 0; l = 0 and l == 03
The auxiliary equation of (1) is
∴ The only non zero eigen value of A is 3 D2 + 2D + 1 = 0
Hence, the correct answer is (3) ⇒ (D + 1)2 = 0
Question Number: 3 Question Type: MCQ ⇒ D = –1 ; –1
∴The general solution of (1) is
The Laplace Transform of f(t) = e sin(5t) u(t) is __________.
2t
y = (c1 + c2 t) e–t                       (3)
5 5 Given y(0) = 1⇒ y = 1 at t = 0
(A) s 2 − 4 s + 29 (B) 2
s +2 ∴ From (3),
1 = (c1 + c2 x 0)e-0
(C) s−2 (D) 5 ⇒ c1 = 1
s − 4 s + 29
2
s+ 5
Also given y(1) = 3e–1 at y = 3e–1 at t = 1
Solution: Given f(t) = e2t sin (5t) u (t) From (3)
5 3e–1 = (c1 + c2 × 1) e–1
We know that L e 2t sin (5t ) = 3e–1 = (1 + c2) e–1
 
( s − 2 )2 + 5 2
GATE 2016 Solved Paper Set − 1  |  39

3e–1 = e-1 + c2 e–1 24 48pi


= 2pi × =
c2 = 2 13 13
Substituting the values of c1 and c2 in (3), Hence, the correct option is (B)
We get
y = (1 + 2t)e–t  (4) Question Number: 6 Question Type: MCQ
y(2) = yat t = 2 = (1 + 2 × 2)e-2 = 5e–2 s
The transfer function of a system is Y ( s) = S + 2 the steady
Hence, the correct option is (B) R( s)
Question Number: 5 Question Type: MCQ state output y(t) is Acos (2t + φ) for the

The value of the integral input cos(2t)the values of A and φ respectively are
1 1
2z + 5 (A) ,–45° (B) ,+ 45°
∫  1 2
dz 2 2
(
 z −  z − 4 z + 5
2
)
(C) 2 ,–45° (D) 2 ,+45°
Over the contour |z|= 1, taken in the anti-clockwise direc- Solution: 
tion, would be
24pi 48pi 24 12 s
(A) (B) (C) (D) R(s) (3)
V(s)
13 13 13 13 s+2
Solution:  Given
Y(t) = Acos(2t + φ)

Y ( s) s
A= =
R( s ) s + 2 w=2

2 1
= =
8 2

w 
& = 90 – tan-1   = 450
 2  w =2
2z + 5 Hence, the correct option is (B)
Let I = ∫ dz
 
C z −

1
2
 z 2
− 4 z + (
5 ) Question Number: 7 Question Type: MCQ
The phase cross-over frequency of the transfer function
Where C is z = 1
G(S) = 100 in rad/s is
2z + 5 ( S + 1)3
Let f(z) = Solution:
 1 2
(
 z −  z − 4 z + 5
2
) Phase of system = − 1800
The singularities of f(z) –3 tan-1 (W) = − 1800
1 W = tan 60 = 3
are z = ; z = 2 ± i
2 Hence, the correct option is (A)
1
of these, only z = lies inside C. Question Number: 8 Question Type: MCQ
2
2z + 5 Consider a continuous-time system with input x(t) and out-
∴Ι =
∫  z − 1 dz
( )
2 put y(t) given by
C   z − 4z + 5
2 y(t) = x(t) cos(t)

( 2 z + 5) ( z2 − 4 z + 5) dz This system is
=
∫  1 (A) linear and time-invariant
C  z − 
2
(B) non-linear and time-invariant
2z + 5 1
2pi. g ( a ) , where g ( z ) = 2 and a = (C) linear and time-varying
z − 4z + 5 2
(D) non-linear and time-varying
(By Cauchy’s Integral Formula)
Solution:  y(t) = x(t) cost
40 | GATE 2016 Solved Paper Set − 1

Scalling : (A)8 (B)10


x1(t) = ax(t) cost (C)12 (D)14
y1(t) = x1(t) cost
= ax1(t) cost Solution:
Ana log output
y1(t) = ay(t) Resolution R = No. of. steps
Additivity 55 − ( −40)
x3(t) = x1(t) cost 0.1°C =
2 −1
n
y3(t) = [x1(t) x2(t)]
= x1(t) cost + x2(t) cost 95
y3(t) = y1(t) + y2(t) 2n – 1 =
0.1
so it is Linear system y(t) = x(t) cost 2n – 1 = 950
y1(t) x1(t) cost 2n = 951
y2(t) x2(t) cost n = 10
Hence, the correct option is (B)
Sub: x2(t) = x1 (t – to) cost
y2(t) = x1 (t – to) cost Question Number: 11 Question Type: MCQ
y1 (t – to) = x1 (t – to) cos(t – to) Consider the following circuit which uses a 2-to-1 multi-
y2 (t) = y1 (t – to) plexer as shown in the figure below. The Boolean expression
It is time-variant system for output F in terms of A and B is
Hence, the correct option is (C)
Question Number: 9 Question Type: MCQ
+∞
The value of ∫−∞
e − t d ( 2t − 2)dt , whered (t ) , is the Dirac delta
function, is
1
(A) (B) 2e
2e
1
(C) 12 (D) (A) A ⊕ B (B) A + B
e 2e 2
(C) A + B (D) A ⊕ B
Solution: Solution:

∫e
−t δ (2t − 2) . dt = ?
−∞

 b  1
d  a (t − )  = δ (t – b/a)
 a  a

∫e 1−t
⇒ .
.δ(t – 1). dt
−∞ 2
F = S I0 + S. I1
1 1
= {e−t}t = 1 = = B . A + B.A
2 2e
Hence, the correct option is (A). = A  B = A⊕ B
Hence, the correct option is (D)
Question Number: 10 Question Type: MCQ Question Number: 12 Question Type: NAT
A temperature in the range of −40˚ C to 55˚ C is to be meas- A transistor circuit is given below. The Zener diode break-
ured with a resolution of 0.1˚ C. the minimum number of down voltage is 5.3 V as shown. Take base to emitter volt-
ADC bits required to get a matching dynamic range of the age drop to be 0.6 V. The value of the current gain β is
temperature sensor is _________.
GATE 2016 Solved Paper Set − 1 | 41

Solution: Question Number: 14 QuestionType: NAT


A soft-iron toroid is concentric with a long straight conduc-
tor carrying a direct current/. If the relative permeability
µ Γ of soft-iron is 100, the ratio of the magnetic flux densi-
ties at two adjacent points located just inside and just out-
side the toroid, is _____.
Solution: a– radius of toroid

M-flux density, B of a toroid which is concentric with a


conductor (N = 1) is given by
µ NI Bin
B=
2 π ρ Bout
= µ r = 100
Hence, the correct answer is 100
Question Number: 15 Question Type: MCQ
RA and RB are the input resistances of circuits as shown
below. The circuits extend infinitely in the direction shown.
Which one of the following statements is TRUE?
10 − 5.3
I1 = = 1mA
4.7 KW
∴ IB = I1 – 0.5mA = 0.5mA
5.3V – 0.6 – 470 IE = 0
470 IE = 4.7V (A) RA=RB (B) RA =RB=0
4.7 (C) RA< RB (D) RB = RA /(1+RA)
IE = A
470 Solution:
IE = 10mA
IE = (1 + β) . 0.5mA
(1 + β) = 20
β = 19
RA
Hence, the correct answer is (18.0 to 20.0) RA = +2
1 + RA
Question Number: 13 Question Type: NAT RA2 − 2 RA − 2 = 0
In cylindrical coordinate system, the potential produced by RA = 1 ± 3
a uniform ring charge is given by φ = f(r, z), where f is con-
 ∴ RA = 2.73W
tinuous function of r and z. Let Ε be the resulting electric

field. Then the magnitude of ∇ × Ε R 1Ω RA
(A) increase with r (B) is 0 Bb

(C) is 3 (D) decrease with z


Solution:  RA
The potential φ and the electric field Ε are functions of r, RB =
1 + RA
and z alone. They are independent of time.

∇ For static e-fields, ∇ × Ε = 0 (Maxwell’s equation)
Hence, the correct answer is (D)
Hence, the correct option is (B)
42 | GATE 2016 Solved Paper Set − 1

Question Number: 16 Question Type: MCQ Solution:


In a constant V/f induction motor drive, the slip at the maxi-
mum torque
(A) is directly proportional to the synchronous speed.
(B) remains constant with respect to the synchronous speed.
(C) has an inverse relation with the synchronous speed.
(D) has no relation with the synchronous speed. Vx
Ι2 =
2
Solution: At max. to square S = r2 V 
XL Ι1 = Ι 2 + Ι 2 = 2Ι 2 = 2  x  = Vx
2
2
V   3  v2
Tmasa   Tmax =   1 = Ι1 (1) + Vx ⇒ 2v = 1 ⇒ Vx = 0.5V
 f  w s  2 x2
I1 = 0.5A
W. K. that, ω sα f , X 2a f Hence, the correct answer is (0.5)
1
⇒ Sα Question Number: 19 Question Type: MCQ
ωs
In a 100 bus power system, there are 10 generators. In a
Hence, the correct option is (C)
particular iteration of Newton Raphson load flow technique
Question Number: 17 Question Type: NAT (in polar coordinates), two of the PV buses are converted to
PQ type. In this iteration,
In the portion of a circuit shown, if the heat generated in 5 Ω (A) the number of unknown voltage angles increases by
resistance is 10 calories per second, then heat generated by two and the number of unknown voltage magnitudes
the 4 Ω resistance, in calories per second, is _____. increases by two.
(B) the number of unknown voltage angles remains
unchanged and the number of unknown voltage mag-
nitudes increases by two.
(C) the number of unknown voltage angles increases by
two and the number of unknown voltage magnitudes
decreases by two.
Solution: (D) the number of unknown voltage angles remains
unchanged and the number of unknown voltage mag-
Let I be the total current
nitudes decreases by two.
Entering in to the circuit
Solution:
10 Total buses = 100
Ι5 = Ι × = 2i / 3
15 Gen. buses = 10
5 ∴ Load buses = 90
Ι 4 = Ι6 = Ι × =I 3
15 At every load bus → v δ must be calculated
3
At every gent bus → Q, δ must be calculated
Given  2Ι  × 5 = 10 i.e., at all buses, ( δ ) must be calculated. So, voltage phase
 3
angle calculations will not change
Ι= 3 during iteration, two of the gen. buses changes to load buses
2
 3  
2 [∴ because of ‘Q ‘violation]
 2  ∴ Total load buses = 92
Heat generated in 4Ω =   ×4 ∴ Voltage magnitude calculation will increase by two
 3 
  Hence, the correct option is (B)
1
= × 4 = 2cal / sec Question Number:20 Question Type: NAT
2
Hence, the correct answer is (2) The magnitude of three-phase fault currents at buses A
Question Number: 18 Question Type: NAT and B of a power system are 10 pu and 8 pu, respectively.
Neglect all resistances in the system and consider the pre-
In the given circuit, the current supplied by the battery, in fault system to be unloaded. The pre-fault voltage at all
ampere, is ___________. buses in the system is 1.0 pu. The voltage magnitude at bus
GATE 2016 Solved Paper Set − 1 | 43

B during a three-phase fault at bus A is 0.8 pu. The voltage Now, we need to calculate Bus ‘ A ‘ voltage
magnitude at bus A during a three-phase fault at bus B, in
when fault is at Bus ‘B’
pu, is ____.
Assume bus (A) voltage is VA
Solution:
1 1 1 1
8= + = +
zB z + z A zB 4 zB + z A

z B = 0.151 ⇒ z A = 0.114, z = 0.604

From fig (2) ⇒


1 VA 1 v
8= + ⇒8= + A
zB z 0.151 0.604

v A = 0.832 pu

Hence, the correct answer is (0.832)

when fault is at Bus ‘A’ Question Number: 21 Question Type: MCQ


1 1 Consider a system consisting of a synchronous generator
10 = +
z A z + zB working at a lagging power factor, a synchronous motor
working at an overexcited condition and a directly grid-
At Bus B, voltage magnitude is 0.8 PU connected induction generator. Consider capacitive VAr to
0.8 1 be a source and inductive VAr to be a sink of reactive power.
∴ Ι2 = =
z z + zB Which one of the following statements is TRUE?
⇒ 0.8 ( z + z B ) = z (A) Synchronous motor and synchronous generator are
sources and induction generator is a sink of reactive
⇒ z = 4 zB power.
(B) Synchronous motor and induction generator are sources
and synchronous generator is a sink of reactive power.
(C) Synchronous motor is a source and induction generator
and synchronous generator are sinks of reactive power.
(D) All are sources of reactive power.
Solution:

Fault at Bus B:
8 = Ιx + Ι y
1 1
8= +
zB z + z A
From equation (2) • Given syn. Generator operations at lagging pf i.e., an
1 1 inductive load should be connected so, machine will
8= + generate reactive power to lagging load
zB z + z A
32zB2 + 8zAzB = 5zB + zA • Syn. Motor is over excited i.e., it is acting like a syn.
Condenser.
From equation (1) 50zAzB + zA
• Induction generator will absorb reactive power of the
From (3) & (4) development of magnetic field.
ZA = 0.76zB Hence, the correct option is (A)
44 | GATE 2016 Solved Paper Set − 1

Question Number: 22 Question Type: NAT


A buck converter, as shown in Figure (a) below, is work-
ing in steady state. The output voltage and the inductor cur-
rent can be assumed to be ripple free Figure (b) shows the
indicator voltage VL during a complete switching interval.
Assuming all devices are ideal, the duty cycle of the buck
converter is _____________.

V-I characteristic of IGBT


(b)

Solution:

TON Period: VL = Vg – Vo = 30V 1


V-I characteristic of diode
TOFF period: VL = –Vo = – 20V 2
(C)
By solving both the equations Solution:
Vg -20 = 30
When Diode is conducting
Vg = 50V
Vo = DVg dv
= 0.01
20 = D (50) dΙ

2 V = 0.01 × 100 = 1V
D= = 0.4
5
Conduction losses = 1 × 100 + 0.7 × 100
Hence the correct answer is 0.4 Total losses = 100 + 70 = 170W
Question Number: 23 Question Type: NAT Hence, the correct option is 170W
A steady dc current of 100 A is flowing through a power Question Number:24 Question Type: NAT
module (S, D) as shown in Figure (a). The V-I characteris-
A 4-pole, lap-connected, separately excited dc motor is
tics of the IGBT (S) and the diode (D) are shown in Figures
drawing a steady current of 40 A while running at 600 rpm.
(b) and (c), respectively y. The conduction power loss in the
A good approximation for the wave shape of the current in
power module (S, D), in watts, is ____________.
an armature conductor of the motor is given by
GATE 2016 Solved Paper Set − 1 | 45

Case (iii)

i = 10 A
Hence, the correct option is (C)
Question Number: 25 Question Type: NAT
If an ideal transformer has an inductive load element at port
2 as shown in the figure below, the equivalent inductance at
port 1 is

Solution:
P = 4, A = 4, Ι a = 40 A N = 600 rpm
Ι a 40
i= = = 10 A
A 4
n n2
(A) nL (B) n2L (C) (D)
L L
Solution:

If the load element was shifted to primary side, the effective


Case (i) equivalent inductance will be Lprim = n2 L
Hence, the correct option is (B)
Question Number: 26 Question Type: NAT
Candidates were asked to come to an interview with 3 pens
each. Black, blue, green and red were the permitted pen
colours that the candidate could bring. The probability that
a candidate comes with all 3 pens having the same colour
is ______.
Solution:
All 3 pens are same colour = 4ways
Case (ii) Two pens are same colour and third pen different colour =
4 × 3 = 12.
All three are of different colour = 4C3 =4
∴Total number of ways of selecting three pens
from four colours pens = 4 + 12 + 4 = 20.
Favourable cases = 4.
4 1
∴ required probability = = = 0.2
20 5
Hence, the correct option is (0.2)
46 | GATE 2016 Solved Paper Set − 1

Question Number: 27 Question Type: MCQ Solution:

∞ Given that A is a 2 × 2 matrix


Let S = ∑ na n where a < 1 the value of a in the range 0
n= 0
And the eigenvalues of A are 1, –2

< a < 1, such that S = 2 a is ____. Let l1 = 1 and l2 = − 2

Solution:  The eigenvectors of A are X1 and X2 corresponding to the


∞ eigenvalues l1 = 1 and l2 = − 2 respectively.
Given S = ∑ na n ; where a < 1
n= 0 The eigenvalues of A2 – 3A + 4I are
Given S = 2a l12 − 3l1 + 4 and l2 − 3l2 + 4
2

∞ i.e., 12 – 3 × 1 + 4 and (-2)2 – 3(-2) + 4 = 2 and 14


⇒ ∑ na n = 2a
n= 0 Also, we know that, A and a matrix polynomial f(A) will
have the same eigenvectors.
⇒ 1 + a + 2a 2 +3a 3 + ...... + ∞ = 2a ∴The eigenvalues and the corresponding eigenvectors of
A2 – 3A + 4I are 2, 14, x1 and x2
a
⇒ = 2a
(1 − a )2 Hence, the correct option is (A)

(∴1 + a + 2a 2 +3a 3 + ...... + ∞ is an AGP with Question Number: 29 Question Type: NAT
Let A be a 4 × 3 real matrix with rank 2. Which one of the
a = 1 ; r = and d =1)
following statement is TRUE?
a
⇒ = 2a (A) Rank of AT A is less than 2.
(1 − a ) 2

(B) Rank of AT A is equal to 2.


1
⇒ (1 − a ) =
2
(C) Rank of AT A is greater than 2.
2
(D) Rank of AT A can be any number between 1 and 3
1
⇒1− a = ± Solution: 
2
1 −1 Given that A is a 4 × 3 real matrix with rank 2
⇒1− a = 1− a =
2 2 i.e., p(A) = 2 (1)
1 1 We know that p (A) = p (AT) = 2
⇒ a = 1− a = 1+
2 2 we have p (ATA) ≤ Min. (p (AT),
⇒ a = 1.7071; a = 0.2929
p (A)) (2
 (2)
⇒ a = 0.2929 (∵ a < 1) [ ∵ p (AB) ≤ Min. (p (A), p (B)]
Hence, the correct option is 0.2929 ∴ (ATA) ≤ 2 (3
 (3)
Question Number: 28 Question Type: MCQ Also, A A is a 3 × 3 real symmetric matrix with 2 linearly
T

independent rows/columns
Let the eigenvalues of a 2 x 2 matrix A be 1, –2 with eigen-
values and eigenvectors x1 and x2 respectively. Then the p (ATA) ≥ 2 (4
 (4)
eigenvectors of the matrix A2 – 3A + 4I would respectively From (3) and (4), )
be
(A) 2, 14; x1, x2 p (ATA) = 2
(B) 2, 14; x1+ x2, x1– x2 Hence, the correct option is (B)
(C) 2, 0; x1, x2 Question Number: 30 Question Type: MCQ
(D) 2, 0; x1+ x2, x1– x2 Consider the following asymptotic Bode magnitude plot (w
is in rad/s).
GATE 2016 Solved Paper Set − 1 | 47

−1
  s o 1 o   1
= [1 1]    −  1
  o s   o 2  
−1
s − 1 0  1
= [1 1]  0 1
 s − 2 

1 1
Y ( s) = + = et + e 2t
s −1 s − 2

at t = loge2

Y ( f ) = e loge + e 2 loge
2 2
Which one of the following transfer functions is best repre-
sented by the above Bode magnitude plot?
= 2 + 22 = 6
(A) 2S
(1+ 0.5S )(1+ 0.25S ) 2 Hence, the correct answer is 6

4(1 + 0.5S ) Question Number: 32 Question Type: MCQ


(B)
S (1 + 0.25S ) Loop transfer function of a feedback system
2S
s+3
(C) is G(s) H(s) = Take the Nyquist contour in
(1 + 2S )(1 + 4 S ) s3 (s − 3)
the clockwise direction. Then, the Nyquist plot of G(s) H(s)
4S
(D) encircles – 1 + j 0.
(1 + 2S )(1 + 4 S ) 2
(A) once in clockwise direction
Solution: (B) twice in clockwise direction
From graph we observe +20 dB/dec slope and 0dB/dec and (C) once in anticlockwise direction
-40dB/dec which say that there exist “1” zero and “3” poles. (D) twice in anticlockwise direction
KS
G ( s) H ( s) =
(1 + ST1 )(1 + ST1 )2 Solution:
RH criteria,
1G (s ) H (s ) = 20 log k + 20 log ω ω = 0.5 = 0dB ⇒ k = 2
s3 – 3s2 + s + 3 = 0
2s
= G ( s) H ( s)
(1 + ST1 )(1 + ST2 )2
→ 0 dB/dec slope occurs at ω 1 which should be greater
than 0.5.
→ -10 dB/dec slope occurs at ω 2 which should be less than 8.
From options choice (A) satisfies all the requirements.
2 sign changes
Hence, the correct option is (A)
2 poles lie right half of s – plane
Question Number: 31 Question Type: NAT
No of Encirclement (N) = z – p
Consider the following state-space representation of a linear
time-invariant system. = 2 – 1 = 1 (+ve)
1
x(t) = 
1 0
 x(t), y(t) = cT x(t), c =   and Hence 1 clock wise encirclement
0 2 1
Hence, the correct option is (A)
1 Question Number: 33 Question Type: NAT
x(0) =   the value of y(t) for t = loge 2 is ______.
1 Given the following polynomial equation
Solution: S3 + 5.5S2 + 8.5S + 3 = 0, the number of roots of the poly-
nomial, which have real parts strictly less than –1 is ______.
Y ( s ) = C ( S Ι − A) B + D
−1
48 | GATE 2016 Solved Paper Set − 1

Solution: Solution:
From the given figure
s= -1 X1(j ω ) = X1(−j ω )
Hence, the correct option is (C)
Put (s – 1) in place of s in the given equation Question Number: 35 Question Type: MCQ
(s – 1)3 + 5.5 (s – 1)2 + 8.5 (s – 1) + 3 = 0 The output of a continuous-time, linear time-invariant sys-
(s – 1)2 [s + 4.5] + 8.5s – 5.5 = 0 tem is denoted by T{x(t)} where x (t) is the input signal.
[s2 + 1 – 2s] [s + 4.5] + 8.5s – 5.5 = 0 A signal z (t) is called eigen-signal of the system T , when
s3 + 4.5s2 + s + 4.5 – 2s2 – 9s + 8.5s – s.5 = 0 T{x(t)} = xz (t), where x(t) is a complex number, in gen-
eral, and is called an eigenvalue of T. Suppose the impulse
s3 + 2.5s2 + 0.5s – 1 = 0 response of the system T is real and even. Which of the fol-
RH criteria lowing statements is TRUE?
(A) cos (t) is an eigen-signal but sin (t) is not
(B) cos(t) and sin(t) are both eigen-signals but with differ-
ent eigenvalues
(C) sin(t) is an eigen-signal but cos (t) is not
(D) cos(t) and sin(t) are both eigen-signals but with identi-
cal eigenvalues
One sign change Solution:
And 2 roots on left of s = –1 Hence, the correct option is (D)
Hence, the correct answer is 2 Question Number: 36 Question Type: MCQ
Question Number: 34 Question Type: NAT The current state QA QB of a two JK flip-flop system is
Suppose x1(t) and x2(t) have the Fourier transforms as shown 00. Assume that the clock rise-time is much smaller than
below. the delay of the JK flip-flop. The next state of the system
is______

(A) 00 (B) 01
(C) 11 (D) 10
CLK JA KA JB KB QA QB
- 1 1 - - 0 0
1 1 1 1 1 1 1
00→11
∴ The next state of the QAQB is 11
Which one of the following statements is TRUE?
Hence, the correct option is (C)
(A) x1(t) and x2(t) are complex and x1(t) x2(t) is also com-
plex with nonzero imaginary part Question Number:37 Question Type: NAT
(B) x1(t) and x2(t) are real and x1(t) x2 (t) is also real 2-bit flash Analog to Digital Converter (ADC) is given
A
(C) x1(t) and x2(t) are complex but x1(t) x2(t) is real below. The input is 0 ≤ VIN ≤ 3 Volts.
The expression for the LSB of the output B0 as a Boolean
(D) x1(t) and x2(t) are imaginary but x1(t) x2(t) is real
function of X2, X1, and X0 is
GATE 2016 Solved Paper Set − 1 | 49

At t = 0+, the rate of change of current through the inductor,


in amperes per second, is.

Solution:
For t < 0,
(A) X0  X 1 ⊕ X 1  (B) X 0  X 2 ⊕ X 1  3
  ΙL = = 1.5 A
2
(C) X0  X 1 ⊕ X 1  (D) X 0 [ X 2 ⊕ X 1 ] At t > 0
Solution X2 X1 X0 B1 B0
Under steady state
 3 3
0 0 0 0 0 Ι L f =  +  = 4.5 A
 1 2
0 0 1 0 1 −t
( 3 2)
0 1 1 1 0 Ι L = 4.5 + (1.5 − 4.5) e

1 1 1 1 1 Ι L( f ) = 4.5 − 3e−2t 3

B0 = X 2 X 1 X0 + X2 X1 X0 d Ι L(t )
dt ( ) −2 ( t )
= −3 −2 e 3
3
B0 = X0 (X1  X2) t =0

= 2 A/s
B0 = X0 ( X 1 ⊕ X 2)
Hence, the correct answer is (2)
Hence, the correct option is (A)
Question Number: 40 Question Type: NAT
Question Number:38 Question Type: NAT
A three-phase cable is supplying 800 kW and 600 kVAr to an
Two electric charges z and −2z are placed at (0,0) and (6,0) inductive load. It is intended to supply an additional resistive
on the x-y plane. The equation of the zero equipotential load of 100 kW through the same cable without increasing
curve in the x-y plane is ________. the heat dissipation in the cable, by providing a three-phase
(A) x = –2 (B) y = 2 bank of capacitors connected in star across the load. Given
(C) x2 + y2 = 2 (D) (x + 2)2 + y2 = 16 the line voltage is 3.3 kV, 50 Hz, the capacitance per phase
of the bank, expressed in microfarads, is ______.
Solution:
x = -2 and y = 2, represent the planes Solution:
P = 800 kW Q = 600KVAR
VL = 3.3 kV
For the given load,
Q
tan θ = = 0.75
P
⇒ cos ϕ = 0.8 lag
Now, current flowing through cable is
P = 3VL Ι L cos ϕ

The point P between the two point charges has the potential, 800 × 10 3
ΙL = = 174.95 A
3 × 3.3 × 10 3 × 0.8
V = 9 × 10 4 − 2 g × 9 × 10 0
9 9
Now, new load of 100 kW is added if the heating in cable
2 4
should be same as before, the current should not alter
Hence, the correct answer is (D) Ι L = 174.95 A, But P = 900 kW
Question Number: 39 Question Type: NAT P = 3VL Ι L cos f = 900 × 103
In the circuit shown, switch S2 has been closed for a long 900 × 103
time. At time t = 0 switch S1 is closed. cos f = = 0.90
3 × 3.3 × 103 × 174.95
50 | GATE 2016 Solved Paper Set − 1

The new Pf requirement is 0.9 lag Question Number: 42 Question Type: MCQ
⇒ sin f = 0.435 If the star side of the star-delta transformer shown in the
The new reactive power from supply should be figure is excited by a negative sequence voltage, then

Q3 = 3 x 3.3 × 103 × 174.95 × 0.435


= 434.98 kVAR
But load requires 600 kVAR
So, the capacitor bank must supply
QC = 600 – 434.98 = 165 kVAR
(A) VAB leads Vab by 60°
165
Each phase = = 55 kVAR (B) VAB lags Vab by 60°
3
Vph2 (C) VAB leads Vab by 30°
= 55 kVAR ⇒ X c = 65.99 (D) VAB lags Vab by 30°
Xc
Solution:
⇒ C = 48.23µF

Hence, the correct answer is (48.23)


Question Number: 41 Question Type: NAT
A 30 MVA, 3-phase, 50 Hz, 13.8 kV, star-connected syn-
chronous generator has positive, negative and zero sequence
reactances, 15%, 15% and 5% respectively. A reactance
(Xn) is connected between the neutral of the generator and
ground. A double line to ground fault takes place involving
phases ‘b’ and ‘c’, with a fault impedance of j0.1 p.u. The
value of Xn (in p.u.) that will limit the positive sequence
generator current to 4270 A is
Solution:
Given a 30 MVA, 3. Ph, 50 Hz, 13.8 kv
Z1 = 0.15, z2 = 0.15, z0 = 0.05 pu
Zf = 0.1 pu, zn = ?;
VAB lags Vab by 300
30 × 106
Ι bare = Hence, the correct answer is (D)
3 ∗ 13.8 × 103
Question Number: 43 Question Type: NAT
Ι a1 ( pu ) = 4270 A : = 1255.1A
A single-phase thyristor-bridge rectifier is fed from a 230 V,
4270 A 50 Hz, single-phase AC mains. If it is delivering a constant
Ι a1 ( pu ) = = 3.4 pu
1255 DC current of 10 A, at firing angle of 30o, then value of the
power factor at AC mains is
Ef (A) 0.87 (B) 0.9
w. r. t Ι a = 1

z1 +
(
z 2 z0 + 3 z f + 3 z n ) (C) 0.78 (D) 0.45
z 2 + z0 + 3 z f + 3 z n Solution:

z1 +
(
z 2 z0 + 3 z f + 3 z n ) = 1.0 Supply power factor = (Distortion factor) × (cos f1 )
z 2 + z0 + 3 z f + 3 z n 3.4
2 2
0.15 +
(0.15)(0.05 + 0.3 + 3zn ) = 1 =
Π
× cos 30 = 0.7796  0.78
0.15 + 0.05 + 0.3 + 3 zn 3.4
Hence, the correct answer is (C)
⇒ zn = 1.08 Question Number: 44 Question Type: NAT
Hence, the correct answer is 1.08 The switches T1 and T2 in Figure (a) are switched in a com-
plementary fashion with sinusoidal pulse width modulation
GATE 2016 Solved Paper Set − 1 | 51

technique. The modulating voltage n m(t) = 0.8 sin (200t)


V and the triangular carrier voltage ( n c ) are as shown in
Figure (b). The carrier frequency is 5 kHz. The peak value
of the 100 Hz component of the load current (iL), in ampere,
is

Solution:
T1 = 1µ sec : T2 = 1µ sec :
(a) 600
V1 (t) = 600v V2 (t ) = 600 − ×t
1 × 10 −6
150
I1(t) = t I1(+) = 100A
1 × 10 −6
1µ sec 1µ sec 1µ sec

E= ∫ v1 (t ) i1 (t ) dt + ∫ v2 (t ) i2 (t ) dt + ∫ v2 (t ) i2 (t ) dt
0 0 0

600 × 150 10 −12 600 × 100 10 −12


E= −6
× + 600 × 100 × 10 −6 − ×
10 2 10 −6 2
= 45 + 60 – 30 = 75 mJ
Hence, the correct answer is (75)
Question Number: 46 Question Type: NAT
A single-phase 400 V, 50 Hz transformer has an iron loss of
(b) 5000 W at the rated condition. When operated at 200 V, 25
Hz, the iron loss is 2000 W. When operated at 416 V, 52 Hz,
Solution: the value of the hysteretic loss divided by the eddy current
Vdc loss is ______.
(V01 )max = ( M .Ι ) = 0.8 × 250 = 200V
2 Solution:
At fundamental frequency V1 = 400v, f1 = 50 Hz W2 = 5000 w

z = R 2 + X 2 = 122 + 16 2 = 20W V2 = 200v, f2 = 25 Hz, W2 = 2000 w


V1 400
(V01 )max 200 = =8
(Ι 01 )max = z
=
20
= 10 A f1 50

v
Hence, the correct answer is (10) = const .
f
V2 200
Question Number: 45 Question Type: NAT = =8
f2 25
The voltage (Vs) across and the current (Is) through a semi-
conductor switch during a turn-ON transition are Wea f 2
shown in figure. The energy sissipated during the turn-
ON transition, in mJ is_______. Wha f
Wi = Af + Bf2
5000 = A(50) + B(50)2
52 | GATE 2016 Solved Paper Set − 1

2000 = A(25) + B(25)2 Solution:


4000 = A(50) + B2(23)2 xd = 0.8 and ∅ = 36.86
xq = 0.6
Wn= 60(52)+0.8((52)
V sin ∅ + I a X a
1000 = B(50)2 – B2(25)2 tan y =
V cos ∅
1000 y = 56.3°
B=
50 − 2 (25)
2 2
y = ∅+δ
B = 0.8 δ = 56.3 – 36.86 = 19.7°
A = 60
E = Vcos δ + Id Xd
Ratio = 60 (52) = 1.44 Id = Ia sin∅ = 0.831
(0.8)(52)2
E = (1).cos(19.7) + (0.831) (0.8)= 1.606V
Hence, the correct answer is (1.44)
Hence, the correct answer is ( 1.606)
Question Number: 47 Question Type: NAT
A DC shunt generator delivers 45 A at a terminal voltage of Question Number: 49 Question Type: MCQ
220 V. The armature and the shunt field resistances are 0.01 A single-phase, 22 kVA, 2200 V/ 220 V, 50 Hz, distribution
Ω and 44 Ω respectively. The stray losses are 375 W. The transformer is to be connected as an auto-transformer to get
percentage efficiency of the DC generator is __________. an output voltage of 2420 V. Its maximum KVA rating as an
auto- transformer is _______.
(A) 22 (B) 24.2
(C) 242 (D) 2420
Solution:
22 KVA
22 × 1000
Ι1 = = 10 A
2200
22 × 10000
Ι2 = = 100 A
220

Vt = 220 v An auto transformer will give maximum VA rating of it op-


erated with a voltage ration near to unity
Ι L = 45 A
Two connections are possible
Ra = 0.01, rsh = 44 1)2200/2420v 2) 220/2420v
But connection (1) is suitable for maximum VA rating
Wstray = 375w
Eg = 220 + (50)(0.01)
Eg.Ia = 220.5 x 50
= 220.5
= 11025 + 375
Pin = 11025 + 375 = 11400
h = 86.84%

Hence, the correct answer is (86.84)


VA rating = (2420)(I2)
Question Number: 48 Question Type: NAT = (2420)(100)
A three-phase, 50 Hz salient-pole synchronous motor has = 242 KVA
a per-phase direct-axis reactance (Xd) of 0.8 pu and a per- Hence, the correct option is (C)
phase quadrature-axis reactance (Xq) of 0.6 pu. Resistance Question Number: 50 Question Type: MCQ
of the machine is negligible. It is drawing full-load current A single-phase full-bridge voltage source inverter (VSI) is
at 0.8 pf (leading). When the terminal voltage is 1 pu, per- fed from a 300 V battery. A pulse of 120o duration is used to
phase induced voltage, in pu, is ________.
GATE 2016 Solved Paper Set − 1 | 53

trigger the appropriate devices in each half-cycle. The rms  3 D12 .D13 .D23 
value of the fundamental component of the output voltage, (1.05)( 2/ ×/ 10
/ / −7/ × 4.85) = 2/ ×/ 10
/ / −7/ In   hfont
in volts, is  0.7788r 
(A) 234 (B) 245 (C) 300 (D) 331 3 D12 .D13 .D23
(1.05)( 4.85) =
0.7788r
Solution:
Fundamental output voltage, D = 1.42 m
4Vdc nπ Hence, the correct answer is (1.42)
v01 = sin ndx sin x sin nωt
nπ 2 Question Number: 52 Question Type: NAT
2 2Vdc
( v01 )rms =
p
× sin d In the circuit shown below, the supply voltage is 10 sin(1000)
volts. The peak value of the steady state current through the
2 2 × 300 1Ω resistor, in amperes, is ______.
= × sin 60 = 234V
p 2µf
Hence, the correct option is (A) 4Ω
500mH
Question Number: 51 Question Type: NAT
250µF 1Ω
Single-phase transmission line has two conductors each of 5Ω
10 mm radius. These are fixed at a center-to-center distance
of 1 m in a horizontal plane. This is now converted to a three- 0.02mH
phase transmission line by introducing a third conductor of Solution:
the same radius. This conductor is fixed at an equal distance
D from the two single-phase conductors. The three-phase 2µf
4Ω
line is fully transposed. The positive sequence inductance 500mH
per phase of the three-phase system is to be 5% more than 250µ 1Ω
that of the inductance per conductor of the single-phase sys- 5Ω
F
tem. The distance D, in meters, is _____.
Solution: 0.02m
H
When a 1 – ph system is available
10sin (1000t)

Under steady state,


Inductive and capacitive

 D Element branches impedance becomes infinity. So they act


L1− ph = 2 × 10 −7 In   H m as open circuit
 D3 
10 sin (1000t )
D → 1mt Ι=
4 +1+ 5
Ds → 0.7788r
Ι = 1sin (1000t )
 1 
L1− ph = 2 × 10 −7 In  H m Ι p = 1Amp
 0.7788 × 10 × 10 −3 
Hence, the correct answer is (1)
= 2 × 10 −7 × 4.85 H m
Question Number: 53 Question Type: NAT
Now a 3 – ph system A dc voltage with ripple is given by ν(t) = [100 + 10 sin w
(t) − 5 sin (3 w (t))] volts.
Measurements of this voltage ν(t), made by moving-coil
and moving-iron voltmeters, show readings of V1 and V2
respectively. The value of V1 – V2, in volts, is _______.
Solution:
Moving coil, V1 = 100V

2 10 2 52
Moving iron, V2 = 100 + +
2 2
54 | GATE 2016 Solved Paper Set − 1

= 100.312V Question Number: 55 Question Type: NAT


V2 – V1 = 0.312V In the circuit shown below, the node voltage VA is_______V.
Hence, the correct answer is (0.312)
Question Number: 54 Question Type: NAT
The circuit below is excited by a sinusoidal source. The
value of R, in Ω, for which the admittance of the circuit
becomes a pure conductance at all frequencies is .
100
R
F

0.02H R Solution:

Solution:
100
R
F

KVL at node A
0.02H R
VA V + 10Ι1 VA − 10
−5+ A + =0
5 5 10
V − 10
R  Ι1 = A
L  10
R2 + + j  w RL −
C  wC  VA V V − 10 VA − 10
z= y −5+ A + A + =0
 1  5 5 5 10
2R + j  w L − 
 wC  1 1 1 1 
VA =  + + +  = 5 + 2 + 1
1 2R  5 5 5 10 
= =
2 R2 + L 8 8 80
VA = = = volts = 11.428volts
c 3 1 6 1 7
+ +
5 10 10 10
1 2R
=
R R 2 + 200 Hence, the correct answer is (11.428)
2 R 2 = R 2 + 200
R 2 − 200
R = 14.14Ω
Hence, the correct answer is (14.14)
GATE 2016 Solutionved Paper
Electrical Engineering
Set – 2
Number of Questions: 65 Total Marks:100

Wrong answer for MCQ will result in negative marks, (-1/3) for 1 Mark Questions and (-2/3) for 2 Marks Question.

General Aptitude
Number of Questions: 10  Section Marks: 15

Q.1 to Q.5 carry 1 mark each and Q.6 to Q.10 carry 2 Choice (D) contradicts the given statements.
marks each.
From the given statements it is clear that R2 D2 is the only
Question Number: 1 Question Type: MCQ which can repair aeroplanes. Hence, (B) can be inferred.
The chairman requested the aggrieved shareholders to Hence, the correct option is (B)
_______ him
Question Number: 5 Question Type: MCQ
(A) bare with (B) bore with
(C) bear with (D) bare If 9 y − 6 = 3, then y – 4y/3 is ______
2

(A) 0 (B) +1/3


Solution: The phrasal verb ‘bear with’ is appropriate in the (C) –1/3 (D) Undefined
given blank. To bear with someone or something is to be
patient with somebody or something. Solution:
|9y – 6| = 3
Hence, the correct option is (C)
1
Question Number: 2 Question Type: MCQ 9y – 6 = –3 ⇒ y =
3
Identify the correct spelling out of the given options: 9y – 6 = 3 ⇒ y = 1
(A) Managable (B) Manageable 4y 1 4 −1 4 −1
\ y2 –
= – = or 1 – =
(C) Mangaeble (D) Managible 3 9 9 3 3 3
4y −1
Solution: Option B as the correct spelling. \ In either case, y2 – = 3 .
3
Hence, the correct option is (B) Hence, the correct option is (C)
Question Number: 3 Question Type: MCQ Question Number: 6 Question Type: NAT
Pick the odd one out in the following
13, 23, 33, 43, 53
(A) 23 (B) 33
(C) 43 (D) 53
Solution: Except 33, rest are all prime numbers.
Hence, the correct option is (B)
Question Number: 4 Question Type: MCQ
R2D2 is a robot, R2D2 can repair aeroplanes. No other
robot can repair aeroplanes.
(A) R2D2 is a robot which can only repair aeroplanes The graph represents the installed capacity for cement pro-
(B) R2D2 is the only robot which can repair aeroplane duction (in tonnes) and the actual production (in tonnes) of
(C) R2D2 is a robot which can repair only aeroplane nine cement plants of a cement company. Capacity utiliza-
(D) Only R2D2 is a robot tion of a plant is defined as ratio of actual production of
Solution: Choice (A) indicates that R2D2 cannot do any- cement to installed capacity. A plant with installed capac-
thing to an aeroplane apart from repairing. This is not in the ity of at least 200 tonnes is called a large plant and a plant
scope of the given statements. with lesser capacity is called a small plant. The difference
between total production of large plants and small plants in
Choice (C) indicates that R2D2 cannot repair any other thing tonnes is ______
except aeroplanes. This is also not in the scope of the given
statements.
56 | GATE 2016 Solved Paper Set – 2

Solution: Plants 1, 4, 8, 9 are large while 2, 3, 5, 6, 7 are Based on the proposal,


small. The total production of the large plants is 160 + 190 (i) In the initial years, SIT students will get degrees
+ 230 + 190, i.e., 770. from IIT
(ii) SITs will have a distinct national objective
The total production of the small plants is 150 + 160 + 120
(iii) SIT like institutions can only be established in
+ 100 + 120, i.e., 650
consolation with IIT
The difference is 120 tonnes. (iv) SITs will serve technological needs of a develop-
ing country.
Hence, the correct answer is (120)
(A) (iii) and (iv) only
Question Number: 7 Question Type: MCQ (B) (i) and (iv) only
A poll of students appearing for masters in engineering (C) (ii) and (iv) only
indicated that 60% of the students believed that mechani- (D) (ii) and (iii) only
cal engineering is a profession unsuitable for women. A
Solution: Neither (i) nor (iii) is in the scope of the passage.
research study on women with masters or higher degrees
in mechanical engineering found that 99% of such women Since, SITs are being established with a specific purpose,
were successful in their professions. (iii) can be inferred.
Which of the following can be logically inferred from the (iv) is a direct extract of the given passage. Hence, only (ii)
above paragraph? and (iv) can be inferred.
(A)  Many students have misconceptions regarding
Hence, the correct option is (C)
various engineering disciplines
(B) Men with advanced degrees in mechanical engineer- Question Number: 9 Question Type: MCQ
ing believe women are well suited to be mechanical Shaquille O’ Neal is a 60% career free throw shooter, mean-
engineers ing that he successfully makes 60 free throws out of 100
(C) Mechanical engineering is a profession well suited attempts on average. What is the probability that he will
for women with masters or higher degrees in me- successfully make exactly 6 free throws in 10 attempts?
chanical engineering (A) 0.2508 (B) 0.2816
(D) The number of women pursuing higher degrees in (C) 0.2934 (D) 0.6000
mechanical engineering is small
Solution: The probability of exactly r free throws in 10 at-
Solution: The given passage brings out the opinion of stu- tempts is given by the (r + 1)th term in the expansion of (0.4
dents with respect to mechanical engineering only. Hence, + 0.6)10, which is 10 Cr (0.4)10 − r (0.6)r
(A) cannot be inferred. The probability of exactly 6 successful throws is
10
C6 (0.4)4 (0.6)6
The statement does not make a specific reference to the
opinions of men with advanced degrees in mechanical engi- 10(9)(8)(7) 256 46656
= = 210 (0.0256)
neering. Hence, (B) cannot be inferred. The research result 2(3)( 4) 10 4 106
indicates that nearly all the women with masters or higher (0.046656) = 0.2508.
degrees in mechanical engineering were successful. Hence, Hence, the correct option is (A)
(C) can be inferred. The passage does not provide any in-
Question Number: 10 Question Type: NAT
formation about the number of women pursuing higher
degrees in mechanical engineering. Hence, (D) cannot be The numeral in the units position of 211870 + 146127 × 3424
inferred. is ______
Hence, the correct option is (C) Solution:
Question Number: 8 Question Type: MCQ The units digit of 211870 is 1
Sourya committee had proposed the establishment of The units digit of 146127 is 6
Sourya Institutes of Technology (SITs) in line with Indian The units digit of 3424 is 1
Institutes of Technology (IITs) to cater to the technological
\ The units digit of the given expression is 1 + (6) (1) = 7
and industrial needs of a developing country.
Hence, the correct answer is (7)
Which of the following can be logically inferred form the
above sentence?
GATE 2016 Solved Paper Set – 2  |  57

ELECTRICAL ENGINEERING
Number of Questions: 55  Section Marks: 85

Q.1 to Q.25 carry 1 mark each and Q.26 to Q.55 carry 2 Solution:
marks each.
Given
Question Number: 1 Question Type: MCQ
The output expression for the Karnaugh map shown below is

(A)
A + B (B)
A+C Transfer the secondary parameters to primary side.
2
(C)
A + C (D)
A+C  
R1 = R  V1 
V 
Solution: 2

A/BC 00 01 11 10
2
 100  80 ×1000
= (80K)   = = 8W
0 1 0 0 1

100 ×100 
100 ×100

1 1 1 1 1

1
2
f=A+ C
Hence, the correct option is (B) X = 40K  
1
= 4W
c
100 
Question Number: 2 Question Type: MCQ
The circuit shown below is an example of a

(A) Low pass filter (B) Band pass filter 100


I =
(C) high pass filter (D) Notch filter 8 + j(10 − 4)

Solution: The given circuit is a practical integrator, so it 100


III = = 10A
acts like a low pass filter. 10
Hence, the correct option is (A) Hence, the correct answer is (10)
Question Number: 3 Question Type: NAT Question Number: 4 Question Type: MCQ
The following figure shows the connection of an ideal trans- Consider a causal LTI system characterized by differential
former with primary to secondary turns ratio of 1: 100. The equation dy (t ) + 1 y (t ) = 3x(t). The response of the system
applied primary voltage is 100V (rms), 50Hz, AC. The rms dt 6 t

value of the current I, in ampere, is ________. to the input x(t) = 3e 3 u (t ) , where u(t) denotes the unit step
function, is _______.
t

(A) 9e 3 u (t )
t

9e 6 u (t )
(B)
58 | GATE 2016 Solved Paper Set – 2

(C)
t t ∂u ∂v
= 2 ≠ (= 0)
− −
9e 3 u (t ) − 6e 6 u (t )
∂x ∂y
t t
(D)

54e 6 u (t ) − 54e 3 u (t )

∴ The real and imaginary parts of f(z) fails to satisfy the
Cauchy – Riemann equations.
Solution: Hence f(z) is not analytic.
Taking Laplace on both sides
Hence, the correct option is (B)
 1
 s +  Y(s) = 3 X(s) Question Number: 7 Question Type: MCQ
 6 
Y (s) A 3 × 3 matrix P is such that, P = P. Then the eigenvalues
3
3
= of P are
X (s) 1
s+ (A) 1, 1, –1
6
    (B) 1, 0.5 + j0.866, 0.5 – j0.866
 3   3  (C) 1, –0.5 + j0.866, –0.5, –j0.866
Y(s) =    
   (D) 0, 1, –1
 s + 1   s + 1 
 6    3 
Solution:
 
  Given that P is a 3 × 3 matrix such that P3 = P.
 54 54 
Y(s) =  − Let l be an eigenvalue of P.
1 1 
s + 6 s + 3 As P is a 3 × 3 matrix and P3 = P
  3
We have λ = λ
Y(t) = 54e−t/6 u(t) − 54 e−t/3 u(t)
⇒ l3 − l = 0
Hence, the correct option is (D)
⇒l (l 2 − 1) = 0
Question Number: 5 Question Type: NAT
⇒ l = 0 ; l 2 −1 = 0
Suppose the maximum frequency in a band-limited sig-
nal x(t) is 5kHz. Then, the maximum frequency in x(t) ⇒ l = 0 ; l = ±1
COS(2000pt), in kHz, is ________. ∴ The eigenvalues of P are 0, 1 and -1
Hence, the correct option is (D)
Solution:
Question Number: 8 Question Type: MCQ
The highest frequency component of x(t) ↑ f = 5KHz{x(t) The Solution of the differential equation, for t > 0, y”(t)
1
Cos(2p x 1000t)} = [ x ( f −1000) + x( f +1000) ] 2y’(t) + y(t) = 0 with initial conditions y(0) = 0 and y’(0) =
2
1, is (u(t) denotes the unit step function),
\The max frequency in the product signal is, f + 100 = (A) te–tu(t) (B) (e–t – te–t) u(t)
5000 + 1000 = 6KHz (C) (–e–t + te–t) u(t) (D) e–t u(t)
Hence, the correct answer is (6) Solution: Given differential equations is
Question Number: 6 Question Type: MCQ y11+ 2y1 + y = 0 (1)
Consider the function f(z) = z + z* where z is a complex with the initial conditions
variable and z* denotes its complex conjugate. Which one y(0) = 0 and y1(0) = 1 (2)
of the following is TRUE? Applying Laplace transform on both sides of (1),
(A) f(z) is both continuous and analytic We have
(B) f(z) is continuous but not analytic
(C) f(z) is not continuous but is analytic L  y11  + 2 L  y1  + L [ y ] = 0
(D) f(z) is neither continuous nor analytic
Solution: (
⇒ s 2 y − sy(0) − y1 (0) + 2 sy − y(0) + y = 0 )
Given f(z) = z + z ∗ where y = L [ y ]
Let z = x + iy ⇒ z ∗ = x – iy
∗ ×00−–11+⇒
⇒ s2 y − s × 2 s y y−−
+2s 2×0×0+0−y+
–2s× 2
+y 02=s y0− 2 × 0 + y = 0
1=
∴ f(z) = z + z = (x + iy) + (x – iy)
⇒ ( s 2 + 2s + 1) y = 1
⇒ f(z) = 2x
Clearly f(z) is continuous 1 1
⇒ y= =
Let f(z) = 2x + i0 = u + iv (s 2
+ 2 s + 1) ( s + 1)
2

∴ u = 2x ; v = 0
Applying the inverse Laplace transform on both sides,
∂u ∂u ∂v ∂u
=2; =0; = 0 and =2
∂x ∂y ∂x ∂x
GATE 2016 Solved Paper Set – 2  |  59

  Solution:
1 
L−1  y  = L−1   Given f(-x) = -f(x)
  2
 ( s + 1)  ∞
  ∑ a
k =1 2 k +1
Sin ( 2k +1) is an odd function.
 1 
⇒ y = L−1   ∴ The Fourier series expansion of f(x) consists of only
 ( s − (−1))2 
  sine terms.

= te−t = te−t u (t ) i.e., f(x) = ∑ b sin (kx)
k

∴ The solution of the given differential equation is k =1

y = te u (t )
−t
Hence, the correct option is (B)
Hence, the correct option is (A) Question Number: 11 Question Type: MCQ
Question Number: 9 Question Type: MCQ A resistance and a coil are connected in series and supplied
The value of the line integral form a single phase, 100V, 50Hz ac source as shown in the
figure below. The rms values of possible voltages across the

∫ (2xy dx + 2x ydy + dz)
c
2 2
resistance (VR) and coil (VC) respectively, in volts, are

along a path joining the origin (0, 0, 0) and the point (1, 1, 1) is
(A) 0 (B) 2
(C) 4 (D) 6
Solution: We have to evaluate the line integral

∫ (2xy dx + 2x ydy + dz)
c
2 2


(A) 65, 35
(C) 60, 90
(B) 50, 50
(D) 60, 80
along a path joining the origin (0, 0, 0) and the point (1, Solution:
1, 1)
Equation of the line joining (0, 0, 0) and (1, 1, 1) is
x −0 y −0 z −0
= = = t , (say)
1− 0 1− 0 1− 0
⇒x=y=z=t
⇒ x = t, y = t and z = t
⇒ dx = dt, dy = dt and dz = dt.
And t varies from t = 0 to t = 1
1
 2 xy 2 dx + 2 x 2 ydy + dz  =
∫ ∫  2(t )(t )dt + 2(t 2 )(t ) dt + dt 
2
 
c t =0


1
Vs2 = (VR + VC cosf)2 + (VCsinf)2
 4t 3 + 1 dt
= ∫  
t =0
Vs2 = V 2 + V C2 + 2V V cosf
R R C
1
= t 4 + t  =2
t =0

Hence, the correct option is (B) V S2 − VR2 − V C2


cosf =
2VR VC
Question Number: 10 Question Type: MCQ
Let f(x) be a real, periodic function satisfying f(–x) = –f(x). From the Choices, Choice (D) gives f = 90°
The general from of its Fourier series representation would be Hence, the correct option is (D)
∞ Question Number: 12 Question Type: NAT
(A) f(x) = a0 + ∑a k =1
k Cos( kx )
The voltage (V) and current (A) across a load are as follows.

(B)
f(x) = ∑b k Sin ( kx ) v(t) = 100Sin(wt),
k =1
I(t) = 10Sin(wt – 60°) + 2Sin(3wt) + 5Sin(5wt)

(C)
f(x) = a0 + ∑a k =1
2k Cos ( kx ) The average power consumed by the load, in W is ________.
∞ Solution: Average power consumed = Vrms . Irms . cosf
(D)
f(x) = ∑
k = 0 +1
a2 k +1Sin ( 2k +1) x
=
1000
×
10
× cos 60° = 250watts
2 2
Hence, the correct answer is (250)
60 | GATE 2016 Solved Paper Set – 2

Question Number: 13 Question Type: MCQ Solution: Given


A power system with two generators is shown in the fig- C = 10nF/Km/Ph
ure below. The system (generators, buses and transmis- L = 1.6mH/Km/Ph
L
sion lines) is protected by six over current relays R1 to R6. \ Surge impedance (ZS) =
C
Assuming a mix of directional and non directional relays at 1.6 ×10−3
appropriate locations, the remote backup relays for R4 are = = 400W
10 ×10−9
( 400 × 103 )2
Load impedance (ZL) =
300 × 106
= 533.33W
As load impedance is more than surge impedance, the re-
(A) R1, R2 (B)
R2, R6 ceiving end voltage will rise more than sending end voltage.
(C) R2, R5 (D)
R1, R6 \ VR VS ⇒ VR > 400KV
Solution: \ We need an inductive compensation to reduce the voltage
to 400KV.
Hence, the correct option is (B)
Question Number: 16 Question Type: MCQ
A parallel plate capacitor filled with two dielectrics is
shown in the figure below. If the electric field in the region
A is 4kV/cm, the electric field in the region B, in kV/cm,
is ____.
* If a fault was taken place b/w (3) & (4) R3 & R4 must
operate. A B
2 cm
* If R4 is failed to operate, then ∈r = 1 ∈r = 4
to limit I1 → R1 must operate
(A) 1 (B) 2
to limit I2 → R6 must operate.
(C) 4 (D) 16
Hence, the correct option is (D)
Solution: For parallel plate capacitor electric field is same
Question Number: 14 Question Type: MCQ
Hence, the correct option is (C)
A power system has 100 buses including 10 generator
buses, for the load flow analysis using Newton – Raphson, Question Number: 17 Question Type: NAT
method in polar coordinates, the size of the Jacobian is A 50MVA, 10KV, 50HZ, star-connected, unloaded three-
(A) 189 × 189 (B) 100 × 100 phase alternator has a synchronous reactance of 1p.u and a
(C) 90 × 90 (D) 180 × 180 sub-transient reactance of 0.2p.u. If a 3-phase short circuit
occurs close to the generator terminals, the ratio of initial
Solution:
and final values of the sinusoidal component of the short
Total buses = 100
circuit current is ________.
Gen buses = 10
\ Load buses = 90 → (m) Solution: Given 50MVA, 10KV alternator
Gen buses = 10 → n ZS = 1.0PU
Size of Jacobian matrix is Z11
s = 0.2PU

= [2m + (n − 1)] [2m – (n − 1)] Immediately after the short ckt (During-transient period)
= [2 × 90 + (10 − 1)] [2 × 90 + (10 − 1)]
= 189 × 189
Hence, the correct option is (A)
Question Number: 15 Question Type: MCQ
The inductance and capacitance of a 400kV, three-phases
50HZ lossless transmission line are 1.6mH/km/phase and 1.0
10nF/km/phase respectively. The sending end voltage is ⇒ ISC = = 1PU
1.0
maintained at 400kV. To maintain a voltage of 400kV at the At steady state
receiving end, when the line is delivering 300MW load, the
shunt compensation required is
(A) Capacitive (B) Inductive
(C) Resistive (D) Zero
GATE 2016 Solved Paper Set – 2  |  61

1.0
⇒ ISC = = 1PU Ι0
1.0 Ripple voltage, DV = .T
C ON
Initial 5
= =5 IoD 2 × 0.6
Final 1.0 ∆v= = −3 = 0.06V
Cf 10 × ( 20 × 10 3)
Hence, the correct answer is (5)
Question Number: 18 Question Type: NAT ∆V 0.06
Vpeak = (Vo)avg + = 36 + = 36.03V
Consider a linear time - invariant system with transfer 2 2
function Peak voltage across switch (VS)peak  = 24 + 36.03V = 60.03V
1
H(S) =
(S + 1) Hence, the correct answer is (60.03)
If the input is Cos(t) and the steady state output is ACos(t + Question Number: 21 Question Type: NAT
a), then the value of A is _________.
For the network shown in the figure below, the frequency (in
Solution: rad/s) at which the maximum phase lag occurs, is ________.
1 1 1
A = |H(s)| = = =
s +1 ω +1
2 2

a = −tan-1(w) = −tan-1(1) = −45°


1
A=
2
Hence, the correct answer is (0.707)
Question Number: 19 Question Type: NAT
 1
A three-phase diode bridge rectifier is feeding a constant 1 + s   s + 1 
DC current of 100A to a highly inductive load. If three - Solution: V 0 = 
( s ) =
 
1 10 s + 1
phase, 415V, 50Hz AC source is supplying to this bridge V in( s) 10 + s
rectifier then the rms value of the current in each diode, in T=1
ampere, is ________. aT = 10 a = 10
2 1 1
Solution: I0 w= w= = 0.316 rad/sec
I 10
Diode current Rms, [ID]Rms = s = 3 T a
2 2 Hence, the correct answer is (0.316)
I
= 0 Question Number: 22 Question Type: MCQ
3
100 The direction of rotation of a single-phase capacitor run
= = 57.735A
3 induction motor is reversed by
Hence, the correct answer is (57.735) (A) Interchanging the terminals of the AC supply
(B) Interchanging the terminals of the capacitor
Question Number: 20 Question Type: NAT (C) Interchanging the terminals of the auxiliary winding
A buck-boost DC – DC converter, shown in the figure below, is (D) Interchanging the terminals of both the windings.
used to convert 24V battery voltage to 36V DC voltage to feed
Solution: Rotating magnetic field directions can be changed
a load of 72W. it is operated at 20KHz with an inductor of
by interchanging the terminals of auxiliary winding
2mH and output capacitor of 1000mF. all devices are con-
sidered to be ideal. The peak voltage across the Solid-state Hence, the correct option is (C)
switch (S), in volt is _______. Question Number: 23 Question Type: MCQ
In the circuit shown below, the voltage and current sources
are ideal. The voltage (Vout) across the current source, in
volts, is ____.

Solution:
Load power, P0 = V0 I0 = 72W
72
I0 = = 2A
36
D D (A) 0 (B ) 5
V0 = × Vdc ⇒ 36 = × 24
1− D 1− D (C) 10 (D) 20
62 | GATE 2016 Solved Paper Set – 2

Solution: Vout = 5[2] + 10 = 20V (A) 1.8V1 + 7.2V2 – V3 (B) 2V1 + 8V2 – 9V3
Hence, the correct option is (D) (C) 7.2V1 + 1.8V2 – V3 (D) 8V1 + 2V2 – 9V3
Question Number: 24 Question Type: MCQ Solution:
The graph associated with an electrical network has 7
branches and 5 nodes. The number of independent KCL
equations and the number of independent KVL equations,
respectively, are
(A) 2 and 5 (B) 5 and 2
(C) 3 and 4 (D) 4 and 3
Solution:
KVL equation = b − n + 1
=7−5+1=3
Apply virtual GND concept
KCL equation = (n − 1) = 5 − 1 = 4
Hence, the correct option is (D) Vx − V 1 + V x − V 2 = 0
Question Number: 25 Question Type: MCQ 1 4
Two electrodes, whose cross-sectional view is shown in the 5Vx = 4V1 + V2 ------(i)
figure below, are at the same potential. The maximum elec-
tric field will be at the point V x − V 3 + V x − V out = 0
1 9
10Vx = 9V3 + Vout ------(ii)
\ from (i) & (ii)
8V1 + 2V2 = 9V3 + Vout
(A) A (B) B Vout = 8V1 + 2V2 − 9V3
(C) C (D) D
Hence, the correct option is (D)
Solution: Question Number: 28 Question Type: MCQ
Hence, the correct option is (A) Let x1(t) ↔ X1(w) and x2(t) ↔ X2(w) be two signals whose
Question Number: 26 Question Type: MCQ Fourier Transforms are as shown in the figure below. In the
( ) ( )
The Boolean expression a + b + c + d + b + c simplifies to figure h(t) = e −2 t denotes the impulse response.
(A) 1 (B) a.b For the system shown below, the minimum sampling rate
(C) a. b (D) 0 required to sample y(t), so that y(t) can be uniquely recon-
structed form its samples, is
Solution:
f = ( a + b + c + d ) + (b + c)
f = (a + b + c + d ) . (b + c)
f = ( a .b. c .d) ( b .c) = 0
Hence, the correct option is (D)
Question Number: 27 Question Type: MCQ
For the circuit shown below, taking the opamp as ideal, the
output voltage Vout in terms of the input voltages V1, V2, and
V3 is ______.
(A) 2B1 (B) 2(B1 + B2)
(C) 4(B1 + B2) (D) ∞
Solution: The product signal x1(t) x2(t) has the highest fre-
quency component of B1 + B2.
\As per the Nyquist principle,
fs = 2[B1 + B2]
Hence, the correct option is (B)
GATE 2016 Solved Paper Set – 2  |  63

Question Number: 29 Question Type: MCQ 4 π


Ι= ×
 Sin2pt  ∞
π 2
The value of the integral 2 ∫   dt is equal to
 pt  ∞
−∞
i.e., 2 ∫ sin 2pt dt = 2
(A) 0 (B) 0.5 −∞ pt
(C) 1 (D) 2
Hence, the correct option is (D)
Solution: ∞
sin 2πt Question Number: 30 Question Type: NAT
Let Ι = 2 ∫ dt
−∞
πt
30. Let y(x) be the Solution of the differential equation
 sin 2pt 

d2y dy
= 22 ∫ dt  − 4 + 4 y = 0 with initial conditions y(0) = 0 and
 0 pt  dx 2 dx
dy
= 1. Then the value of y(1) is ________.
 sin 2pt  dx x=0
∵ is aneven function
 pt 
Solution: Given differential equation is
d2y dy
4 sin 2πt

− 4 + 4 y = 0 (1)
∴Ι = ∫ dt (1)
(1) dx 2 dx
π 0 t
sin 2πt With the initial conditions
Consider the Laplace transform of dy
t y (0) = 0 and = 1 (2)
dx x= 0

Applying Laplace transform on both sides of (1),
i.e.,L  sin 2pt  =
 t  ∫ L[sin 2pt ]ds d2y  dy 
s L  2  − 4 L   + 4 L[ y ] = 0
 dx   dx 
 
  f (t )  ∞ 
∵ L   = ∫ L  f (t ) ds
  t  s  ⇒ s 2 y − sy(0) − y ’(0) − 4 ( sy − y(0)) + 4 y = 0


where y = L [ y ]
2p
=∫ ds ⇒ s2 y − s × 0 −1 − 4 s y + 0 + 4 y = 0
s + ( 2p )
2 2
s
⇒ ( s 2 − 4 s + 4) y = 1

 s 
= Tan−1   1 1
 2p  ⇒ y= =
s
s 2 − 4 s + 4 ( s − 2)2
 s 
= Tan −1∞ − Tan −1  
 2p  Applying inverse Laplace transform on both sides

p  
= − Tan −1 ( s 2p ) L−1  y  = L−1 
1 
2   2 
 ( s − 2) 
 sin 2pt   s 
∴ L  = cot −1  
 t   2p  ⇒ y = xe 2 x

The solution of (1)
⇒isy = xe 2 x

 sin 2pt   s 
⇒ ∫ e−st   dt = cot −1  
 t   2p  Now y(1) = yat x=1 = 1× e = e2
2×1

Taking s = 0 on both sides, ∴ y(1) = 7.389


Hence, the correct answer is 7.389

 sin 2pt  0
∫e
−oxt   dt = cot −1   Question Number: 31 Question Type: NAT
 t   2p 
0
The line integral of the vector field F =
∞  sin 2pt 
⇒ ∫   dt = cot−1 (0) 5 xziˆ + (3 x + 2 y) ˆj + x zkˆ along a path from (0, 0, 0) to (1,
2 2

 t 
0
1, 1) parameterized by (t, t2, t) is _________.
∞  sin 2pt  p
⇒ ∫   dt = (2) Solution: We have to find the line integral of the vector field
 t  2 (2)
0
F = 5 xzi + (3 x 2 + 2 y) j + x 2 zk
Substituting (2) in (1), we get
64 | GATE 2016 Solved Paper Set – 2

along a path from (0, 0, 0) to (1, 1, 1) parameterized by ⇒ a = 3x + y and b = x+ 3y


(t, t2, t) From (1), a2 + b2 = 1
i.e., along the path ⇒ (3x + y)2 + (x + 3y)2 =1
x = t, y = t2 and z = t 9x2 + 6xy + y2 + x2 + 6xy + 9y2 = 1
⇒ dx = dt, dy = 2t dt and dz = dt 10x2 + 12xy + 10y2 = 1
and t varies from t = 0 to t = 1 ⇒ 8(x + y )2 + 2(x – y)2 = 1
( x + y)2 ( x − y)2
∴ The required line integral is + =1
1/ 8 1/ 2
(1,1,1) (1,1,1) Which represents an ellipse a < b
∫ ∫ (5 xzi + (3x )(
+ 2 y) j + x 2 zk . dxi + dy j + dzk )
2
F .dr = \ Major axis is x + y = 0 and minor axis is x – y = 0.
( 0,0,0) ( 0,0,0)
Hence, the correct option is (D)
(1,1,1)
5 xzdx + (3 x 2 + 2 y) dy + x 2 zdz  Question Number: 33 Question Type: MCQ
= ∫  
( 0,0,0) Let the probability density function of a random variable,
X, be given as:
1
3 −3 x
∫ 5(t )(t )dt + (3(t ) + 2(t 2 )) 2tdt + (t ) 2 (t )dt  e u ( x) + ae 4 x (−x)
2
= fx(x) =
 2
t =0
Where u (x) is the unit step function
1 Then the value of ‘a’ and Probe {X ≤ 0}, respectively, are

∫ 5t + 10t 3 + t 3  dt
2
= 1 1
t =0 (A)
2, (B)
4,
2 2
1 1 1
(C)
2, (D)
4,
∫ 5t + 11t 3  dt
2
= 4 4
t =0
1 Solution: Given the probability density function of a ran-
5 11  dom variable X, is
= t3 + t4
3 4  t = 0 −3 x 3 4x
5 11
fX(x)= e u( x ) + ae u(−x )
2
= +
3 4
 ae 4 x ; −∞ < x < 0
=
53
= 4.4167 i.e., f X ( x ) =  3 −3 x
12  e ; 0≤ x <∞
 2
Hence, the correct answer is (4.4167)

Question Number: 32 Question Type: MCQ
we know that ∫ f X ( x )dx = 1
−∞
3 1  x
Let P =   . Consider the set S of all vectors   such that

1 3  y
0 ∞
3 −3 x
⇒ ∫ ae 4 x dx + ∫ e dx = 1
a  x  2
a2 + b2 = 1 where   = P   Then S is ________.
−∞ 0

b   y ∞
3 e−3 x 
0
a 
⇒ e4 x  + =1
(A) a circle of radius 10 4  −∞ 2 −3  0
1 a 1
(B) A circle of radius ⇒ + =1
1 10 4 2
(C) an ellipse with major axis along   ⇒a=2
1
0
1
(D) an ellipse with minor axis along   And P ( X ≤ 0) = ∫ f X ( x )dx
1 −∞
Solution:
3 1 0
Given P = 1 3
∫ ae
4x
= dx
  −∞
0
Given a2 + b2 =1 ------- (1) e4 x  1
=2  =2
 a  x 3 1  x  a 3 x + y 4  −∞
And   = P   =      = 
b  y 1 3  y b  x + 3 y Hence, the correct option is (A)
 
GATE 2016 Solved Paper Set – 2  |  65

Question Number: 34 Question Type: NAT 3


I1 = − I2 (3)
The driving point input impedance seen from the source Vs 2
Substitute (3) in (a)
of the circuit shown below, in W is_________.
 −2 
20 = 50I1 + 60  I1 
 3 
20 = 50I1 − 40I1
10I1 = 20
I1 = 2
I2 = −4/3
Solution: 2
 
P RC =  4  × 20 = 16 × 20 = 35.55W
 3  9

Hence, the correct answer is (35.55)


Question Number: 36 Question Type: NAT
In the balanced 3-phase, 50Hz, circuit shown below, the
V1 value of inductance (L) is 10mH. The value of the capaci-
IS =
2 tance (C) for which all the line currents are zero, in millifar-
KVL at 3W Resistor ads, is ________.
V − Vs V V
+ − 4V1 + = 0
2 3 6
3V − 3VS + 2V − 24V1 + V = 0
6V − 3VS − 24V1 = 0
V = − V1 + VS
−6V + 3VS − 24V1 = 0
3Vs = 30V1
Vs = 10V1
Vs = 10[2IS] Solution:
V
Zin = S = 20W
ΙS
Hence, the correct answer is (20)
Question Number: 35 Question Type: NAT
35. The Z-parameters of the two port network shown in
the figure are Z11 = 40W, Z12 = 60W, Z21 = 80W , and Z22 =
100W. The average power delivered to RL = 20W, in watts, is
_________.

C
L/3

Solution: L/3
L/3
V 
 1  =  Z11I1 + Z12 I 2  C C
V  Z I + Z + I 
 2  12 1 22 2

V2 = −I2[R2] = −20I2 (1)


20 = I2[10] + V1 Given, IL = 0 ⇒ capacitor must supply
20 = I2 (10) + 40 I1 + 60 I1 inductor current
20 = 50I1 + 60 I2 (a)
V ph
V2 = 80I1 + 100 I2 (2) IC = IL Vph (jwc) =
From (1)
jw L( )
3
−20 I2 = 80I1 + 100 I1 3 3
wc = ⇒ w2 =
−120 I2 = 80I1 wL LC
66 | GATE 2016 Solved Paper Set – 2

3 Solution:
⇒C= = 3mF
10 ×10−3
Hence, the correct answer is (3)
Question Number: 37 Question Type: NAT
In the circuit shown below, the initial capacitor volt-
age is 4V. switch S1 is closed at t = 0. The charge (in
mC) lost by the capacitor form t = 25mS to t = 100mS
is ________.

Without fault:

Solution:
VC(t) = Vf + (Vi − Vf) e-t/T (1.0) (1.0) (1.0(1.0)
Pmax = = = 2PU
VC(t) = 0 + (4 − 0) e−t / 25×10−6 Xq j 0.5

4
During the fault:
VC(t) = 4 e−t ×10 ×4
During the fault:
Q = cvc(t)
= (5 × 10-6) [4e-(t×4×104)]
Charge lost from t = 25ms to t = 100ms
Q = 5 × 10–6× 4 [e–1 − e–4]
= 6.99mC
Hence, the correct answer is (6.99)
Question Number: 38 Question Type: NAT
The single line diagram of a balanced power system is
shown in the figure. The voltage magnitude at the generator
internal bus is constant and 1.0p.u. the p.u reactances of
The above circuit can be drawn as follows.
different components in the system are also shown in the
figure. The infinite bus voltage magnitude is 1.0p.u. A three
phase fault occurs at the middle of line 2.
The ratio of the maximum real power that can be trans-
ferred during the pre - fault condition to the maximum real
power that can be transferred under the faulted condition
is ______
GATE 2016 Solved Paper Set – 2  |  67

1.15 1.0PU T −2
1.0PU \ 0<K<
T −2
Hence, the correct option is (C)

Question Number: 40 Question Type: NAT
0.6656 0.221
At no load condition, a 3-phase, 50Hz, lossless power trans-
mission line has sending-end and receiving-end voltages of
400KV and 420KV respectively. Assuming the velocity of
(1.0)(1.0) travelling wave to be the velocity of light, the length of the
Pmax = = 0.869PU
1.15 line, in km, is ________.
P pre fault 2
The ratio = = = 2.3 Solution:
P during fault 0 . 869
coshnL = A = v S
Hence, the correct answer is (2.3) v R no load

Question Number: 39 Question Type: MCQ YZ 400


1+ =
The open loop transfer function of a unity feedback control 2 420
system is given by
( jωc)( jωc) 400
K (S + 1) 1+ =
G(S) = , K > 0, T > 0 2 420
S (1 + Ts)(1 + 2S )
2
400
The closed loop system will be stable if, 1 − w LC =
4 ( K + 1) 2 420
(A)
0 <T <
K −1 20 ω 2 LC
4 (T + 2) =
(B)
0<K < 420 2
T −2
T +2 1 ω 2 2
(C)
0<K < =
T −2 21 2U 2
8( K + 1)
(D)
0 <T < 21 2 2
K −1 υ2 = w l
2
Solution:
21
1 + G(S) = 0 υ = wl
2
S(1 + TS) (1 + 2S) + KS + K = 0
(2T)S3 + (2 + T)S3 + (1 + K)S + K 3×108
RH criteria ℓ = = 294.847Km
21
314 ×
S3 2T 1+K 2
S2
2+T K Hence, the correct answer is (294.847)
Question Number: 41 Question Type: NAT
S1 ( 2 + T ) (1 + K ) − 2TK 0
2+T
The power consumption of an industry is 500kVA, at 0.8 p.f lag-
ging. A synchronous motor is added to raise the power factor of
S0 K 0 the industry to unity. If the power intake of the motor is 100kW,
the p. f. of the motor is ________.
First column elements should be positive for the system is
to be stable. Solution:
Given T > 0, hence 2 + T > 0 Load:
2TK 500KVA, 0.8Pf lag
(1 + K) >
2+T PL = (500) (0.8) = 400 KW
K>0 QL = (500) (0.6) = 300 KW
 2T  * Now a syn. Motor is added in parallel to rise the Pf to
1 > K  − 1
 unity.
 2 + T 
* So, motor must supply the total load reactive power
 2T − 2 − 7 
1>K  ; \ Qmotor = 300 KVAR
 2+T 
 
Qmotor = 100 KW
T − 2 1 T + 2
1 > K  ;

> P motor
T + 2  K T − 2 Pf of the motor =
Pm + Q m
2 2
68 | GATE 2016 Solved Paper Set – 2

100
= = 0.316 lead
100 + 300
2 2

Hence, the correct answer is (0.316)


Question Number: 42 Question Type: NAT
The flux linkage (l) and current (i) relation for an electro-
magnetic system is l = ( i ) /g. when i = 2A and g(air - gap
length) = 10 cm, the magnitude of mechanical force on the
moving part, in N, is ________.
Solution: VA rating = (300)(20) = 6000
Hence, the correct answer is (188)    = 6KVA
Question Number: 43 Question Type: NAT Hence, the correct answer is (6)
The starting line current of a 415V, 3-phase, delta connected Question Number: 45 Question Type: NAT
induction motor is 120A, when the rated voltage is applied A full-bridge converter supplying an RLE load is shown
to its stator winding. The starting line current at a reduced in figure. The firing angle of the bridge converter is 120°.
voltage of 110V, in ampere is ________. The supply voltage nm(t) = 200pSin(100pt) V, R = 20W, E
Solution: = 800V. The inductor L is large enough to make the output
V current IL a smooth dc current. Switches are lossless. The
The starting current of a 3-ph IM is Ist =
real power feedback to the source, in kW, is __________.
R + Xq
2 2
q

Ist a V

I1 V1 V   110 
= ⇒ I2 =  1  I1 =   (120)
I 2 V2 V2   415 

I2 = 31.80A
Hence, the correct answer is (31.8)
Question Number: 44 Question Type: NAT
A single-phase, 2kVA, 100/200V transformer is recon-
nected as an auto-transformer such that its kVA rating is
maximum. The new rating, in kVA is _________.

Solution:

Solution:
Output voltage, V0 = −E + I0R
V0 = −800 + I0 × 20
2Vm
V0 = cosa
p
KVA = 2KVA 2 × 200p
V0 = cos120
V1 = 100V, V2 = 200V p
V0 = −200V
2×1000
I1 = = 20 A From equation (1) −200 = −800 + I0 × 20
100
I0 = 30A
2×1000
I2 = = 10 A Power feedback to the source, P0 = V0I0
200 = 200 × 30 = 6kW
If it is connected as an auto transformer
Hence, the correct answer is (6)
GATE 2016 Solved Paper Set – 2  |  69
10
Question Number: 46 Question Type: NAT = 0.1 × = 3.513A
0.9
A three-phase Voltage Source Inverter (VSI) as shown in
Hence, the correct answer is (3.51)
the figure is feeding a delta connected resistive load of 30W/
phase. If it is fed from a 600V battery, with 180° conduction Question Number: 48 Question Type: NAT
of solid-state devices, the power consumed by the load, in A single-phase bi-directional voltage source converter
kW, is ________. (VSC) is shown in the figure below. All devices are ideal.
It is used to charge a battery at 400V with power of 5kW
from a source Vs = 220 V(rms), 50HZ sinusoidal AC mains
at unity p.f. If its AC side interfacing inductor is 5mH and
the switches are operated at 20KHz, then the phase shift (d)
between AC mains voltage (Vs) and fundamental AC rms
VSC voltage (Vc1), in degree, is ________.

Solution: Output power, Po = 3 VL IL cosf


V 2 2V 2
= L = dc
R 3R
2 × (600 )
2

= 3 10 = 24kW
×
Hence, the correct answer is (24) Solution:
Question Number: 47 Question Type: NAT PDC = 5000W
A DC - DC boost converter, as shown in the figure below, is PAC = Vsr1 Isr1 cosq = 220 × Isr1 × 1
used to boost 360V to 400V, at a power of 4kW. All devices PAC = PDC
are ideal. Considering continuous inductor current, the rms 5000
current in the Solid state switch (S), in ampere is _______ Isr =
220
Solution: Xs = wL = 100p × 5 × 10-3
I sr x s

sind = d = 9.21°
Vs
Hence, the correct answer is (9.21)
Question Number: 49 Question Type: MCQ
Consider a linear time invariant system x = Ax, with initial
condition x(0) at t = 0. Suppose a and β are eigenvectors
of (2 × 2) matrix A corresponding to distinct eigenvalues
Output voltage, V0 = V dc l1 and l2 respectively. Then the response x(t) of the system
1− D due to initial condition x(0) = a is
360
400 = eλ t β
(A) eλ t α (B) 2
1− D 1

D = 0.1 (C) e 2 α (D)


λ t
eλ t α + e β
λ t 1 2

P0 = V0I0
4000 = 400 × I0 Solution:
I0 = 10A x(t) eAt x(0)
10 10
(IL)avg = I 0 = = A l1 0
1− D 1 − 0.1 0.9 A =  0 l  and x(0) = a
 2
l1 
2 2 e 0
(IL)Rms = (I )avg + (I ripple)Rms
L L
EAt = L-1(SI – A)–1 = 
 l2 

0 e 
 
DI L
(IL)ripple Rms = l1 t 
2 3 e 0  a 
X(t) eAt x(0) =    
By neglecting ripple current, IL(ripple rms) = 0  l2 t 
 
0
 e   0 
10
(IL)Rms = (IL)avg = A
0.9 X(t) = eλ t α 1

(ISW)Rms = D (IL)Rms Hence, the correct option is (A)


70 | GATE 2016 Solved Paper Set – 2

Question Number: 50 Question Type: MCQ Question Number: 52 Question Type: MCQ
A second-order real system has the following properties: The gain at the breakaway point of the root locus of a unity
(a) the damping ratio d = 0.5 and undamped natural feedback system with open loop transfer function G(S) =
frequency wn = 10rad/s KS
is
(b) the steady state value of the output, to a unit step (S −1)(S − 4)
input, is 1.02. (A) 1 (B) 2
The transfer function of the system is (C) 5 (D) 9
(A) 2 1.02
S + 5S + 100 Solution:
KS
(B) 2
102 G(S) =
( S − 1) ( S − 4)
S + 10 S + 100
CE = 1 + G(S) = 0
(C) 100
S 2 + 10 S + 100 ⇒ (S − 1) (S − 4) KS = 0
102
(D)
S 2 + 5S + 100
( S − 1) ( S − 4)
Solution: K=−
S
Second order characteristic equation is S2 + 2 gwnS + wn = 0 dK
For the given data the characteristic equation is S2 + 10S + Break away point occurs at a point =0
dS
100 = 0.
102
For option (B) DC gain = = 1.02 dK  S ( 2S − 5) − (S2 − 5S + 4) 
100 =−
dS =0
Hence, the correct option is (B)  S2 
Question Number: 51 Question Type: MCQ 2S2 – 5S = S2 – 5S + 4
Three single-phase transformers are connected to from S = ±2
a delta-star three-phase transformer of 110kV/11kV. The
transformer supplies at 11kV a load of 8MW at 0.8p.f. lag- K = – (2 − 1) (2 − 4) = 1
2
ging to a near by plant. Neglect the transformer losses. The
ratio of phase currents in delta side to star side is ________. Hence, the correct option is (A)
Question Number: 53 Question Type: NAT
(A) 1 : 10 3 (B)
10 3 : 1
Two identical unloaded generators are connected in par-
(C) 1 : 10 (D) 3 : 10 allel as shown in the figure. Both the generators are hav-
Solution: D / Y connection [110 KV/11KV] ing positive, negative and zero sequence impedances of
j0.4pu., j0.3pu., and j0.15pu., respectively. If the pre-
load on Y side
fault voltage is 1pu, for a line - to - ground (L – G) fault
11KV , 8mv, 0.8 Pf log
at the terminals of the generators, the fault current, in pu.,
⇒ D side MVA rating is also 10MVA is _________.
D side line current will be
10 mvA = 3 * 110* 103* IL
10 ×106
IL = = 52.48A
3 ×110 ×103
52.48
⇒ Iph = = 30.30A
3
Y side:
Solution: Two generators in parallel
10MVA = 3 * 11 × 103* IL
10 ×106
⇒ IL = = 524.86A
3 ∗ 11×103

⇒ Iph = IL = 524.86A
Ι ph(∆) 30.30
The ratio = = 0.0577
Ι ph(Y ) 524.86
1 * Equivalent positive seq. impedance.
=
10 3 (0.4)(0.4)
Z1eq = = 0.2PU
Hence, the correct option is (A) 0.8
GATE 2016 Solved Paper Set – 2  |  71

20 × 60 × 60
Load power (in kW) = = 2kW
1200 × 30
Hence, the correct answer is (2)
Question Number: 55 Question Type: NAT
* Equation negative sequence impedance. A rotating conductor of 1m length is placed in a radially
(0.3)(0.3) outward (about the z - axis) magnetic flux density (B) of 1
Z2eq = = 0.15PU
0.6 Tesla as shown in figure below. Conductor is parallel to and
at 1m distance from the z - axis. The speed of the conductor
in r.p.m. required to induce a voltage of 1V across it, should
be ________.

* Equation zero sequence impedance is


Z0eq = 0.15PU
( ∵ 1st generator neutral is disconnected form ground)

\ Ι a1 = Eg 1.0
= = 2 PU
0.2 + 0.15 + 0.15
Z 1eq + Z 2 eq + Z 0 eq
If = 3Ia1 ( ∵ L – G fault)
= (3) (2) = 6PU Solution: E = BlV
1=1×1×υ
Hence, the correct answer is (6) υ=1
Question Number: 54 Question Type: NAT rw = 1
54. An energy meter, having meter constant of 1200 revo- w=1
lutions/kWh, makes 20 revolutions in 30 seconds for a con- 2pN
=1
stant load. The load, in kW, is ________. 60

Solution:
(
N = 60 2p rpm )
rev = 9.54rpm
Energy meter constant, K = 1200
kWh
Hence, the correct answer is 9.54

S-ar putea să vă placă și